You are on page 1of 53

Practice Questions

B1 C8 II
A physical therapy plan of care for a child with spastic cerebral palsy who is three years old chronologically and cognitively
but at a six month-old gross developmental level would include:
A. reaching for a black and white object while in the supine position.
B. reaching for a multicolored object while in an unsupported standing position.
C. visually tracking a black and white object held nine inches from his face.
D. reaching for a multicolored object while in an unsupported, guarded sitting position.
B2 C1 II
A patient is receiving grade III mobilizations to regain normal midthoracic extension. After three sessions he complains of
localized pain that persists for greater than 24 hours. The therapists treatment would:
A. change to grade II mobilizations to reduce the joint and soft tissue irritation.
B. change to grade IV mobilizations to stretch through the pain.
C. change to self-stretching activities because the patient does not tolerate mobilization.
D. continue with grade III mobilizations followed by a cold pack to the thoracic spine.
B3 C7 II
A therapist wishes to use behavior modification techniques as part of a plan of care to help shape the behavioral responses
of a patient recovering from traumatic brain injury. The BEST form of intervention is to:
A. allow the patient enough time for self-correction of the behavior.
B. encourage the staff to tell the patient which behaviors are correct and which are not.
C. reprimand the patient every time an undesirable behavior occurs.
D. use frequent reinforcements for all desired behaviors.
B4 C2 III
Which is NOT a useful intervention for a patient with multiple sclerosis who presents with a primary deficit of dysmetria?
A. isokinetic training using low resistance at fast movement speeds.
B. PNF patterns using carefully graded resistance and slow reversals.
C. pool therapy using moderate water temperatures.
D. weight cuffs to distal extremities during functional training.
B5 C6 III
A patient suffered fractures of C4 and C5 following trauma received in a motor vehicle accident. Maximum stabilization of
his cervical spine can BEST be achieved with a:
A. four-poster orthosis.
B. halo orthosis.
C. Milwaukee orthosis.
D. soft collar.
B6 C3 II
A patient had been on oxygen, but it was discontinued by physician order yesterday. During physical therapy, the patient
becomes short of breath and requests supplemental oxygen. The patients SaO2 is measured at 90%. The decision that is
NOT appropriate is:
A. administer supplemental O2.
B. allow the patient to rest.
C. continue to monitor SaO2.
D. encourage an efficient breathing pattern.
B7 C2 I
A patient presents with weakness and atrophy of the biceps brachii resulting from an open fracture of the humerus. The
therapists examination includes needle electromyography of the biceps. The muscle response anticipated after the needle is
inserted and prior to asking the patient to contract the muscle is:
A. electrical silence.
B. fibrillation potentials.
C. interference patterns.
D. polyphasic potentials.

333687839.doc

Practice Questions

B8 C4 II
A new staff physical therapist on the oncology unit of a large medical center receives a referral for strengthening and
ambulation for women with ovarian cancer. She is undergoing radiation therapy following a surgical hysterectomy. Her
current platelet count is 17,000. The MOST appropriate treatment activity for this patient at this time is:
A. passive ROM exercise.
B. progressive stair climbing using weighted belts.
C. resistance training at 30% one repetition max.
D. resistance training at 50% one repetition max.
B9 C1 II
An office worker, who is a long-term smoker now with emphysema, is referred to physical therapy. He complains of
increased pain and tingling in both hands after sitting at his desk for longer than one hour. The MOST effective physical
therapy intervention is:
A. mobilization of the first rib and stretching of middle trapezius muscle due to long-term postural changes.
B. strengthening of the upper trapezius and pectoralis major muscles for early thoracic outlet syndrome symptoms.
C. stretching of scalenes and sternocleidomastoid muscles for early thoracic outlet syndrome symptoms.
D. stretching of the pectoralis major and rhomboid muscles due to long-term postural changes.
B10 C8 II
A patient with diabetes is exercising. He reports feeling weak, dizzy, and somewhat nauseous. The therapist notices that he
is sweating profusely and is unsteady on his feet.. The therapists immediate course of action would be to:
A. call for emergency services as he is having an insulin reaction.
B. have a nurse administer an insulin injection for his developing hyperglycemia.
C. immediately administer orange juice for his developing hypoglycemia.
D. insist that he sit down until his orthostatic hypotension resolves.
B11 C2 II
A patient with postpolio syndrome has started attending an aerobic exercise program as an outpatient. When she does not
show up for her scheduled session, the therapist telephones to find out what is wrong. She says that she is very tired and has
not been able to get out of bed for the last two days. Her muscles ache and she is afraid to continue with the exercise class.
The therapists BEST course of action regarding exercise is to:
A. decrease the frequency to once a week for an hour session, keeping the intensity moderate.
B. decrease the intensity and duration but maintain a frequency of 3 times/week.
C. discharge her from the program since exercise is counterproductive in postpolio syndrome.
D. reschedule her exercise workouts for early morning when she is less fatigued.
B12 C7 III
A physical therapy student is on a final clinical rotation. The supervising physical therapist becomes aware that the student
tends to process information all at once, not in an ordered step-by-step manner. The therapists best strategy to ensure
adequate learning for this student is to:
A. focus the student on learning important relationships and concepts.
B. focus the student on objective information and interrelationships.
C. provide real-life examples that link learning to personal experiences.
D. redirect the student to process information in a step-by-step manner.
B13 C7 II
A 69 year-old falls and breaks his hip. He undergoes a total hip replacement at the local medical center. The physician
orders physical therapy two days after surgery. The patient is covered by Medicare and is still in an acute care facility. The
appropriate frequency of treatment would be:
A. daily.
B. every other day.
C. three to four times per week.
D. two times per day, 4 days per week.

333687839.doc

Practice Questions

B14 C3 I
A patient is taking a drug from the sympathomimetic group, albuterol (Proventil). What is the MOST important effect of
this medication?
A. enhance bronchial smooth muscle contraction and airway resistance.
B. increase airway resistance and exercise-induced bronchial dilation.
C. reduce airway resistance and exercise-induced bronchospasm.
D. reduce bronchial constriction and hypertension that accompanies exercise.
B15 C6 III
A physical therapist decides to exercise a patient with fibromyalgia using aquatic therapy. The patient is moderately
deconditioned from months of inactivity and has a history of a mild myocardial infarction 4 years ago. Hydrostatic pressure
exerted by the water can be expected to:
A. increase cardiovascular demands at rest and with exercise.
B. increase resistance as speed of movement increases.
C. provide joint unloading and enhance ease of active movement.
D. reduce effusion and assist venous return.
B16 C6 II
A therapist determines a patient is walking with a backward trunk lean as he takes full weight on his right leg. He also
demonstrates great difficulty going up ramps. The BEST intervention to remediate his problem is to:
A. strengthen hip extensors through bridging.
B. strengthen knee extensors with weights, using 80% one repetition maximum.
C. stretch hip abductors through sidelying positioning.
D. stretch hip flexors through prone lying.
B17 C4 II
If a patient has developed a thick eschar secondary to a full thickness burn, the antibacterial agent MOST effective for
infection control is:
A. nitrofurazone.
B. Panafil.
C. silver nitrate.
D. Sulfamylon.
B18 C8 I
An 89 year-old has gradually lost much of his functional vision over the last 4 years. He complains his vision is "foggy"
and he has difficulty reading. He mistakes images directly in front of him, especially in bright light. He does better walking
across a room and is able to locate items in his environment with his peripheral vision when items are located to both sides.
Based on his descriptions, the therapist suspects he may be presenting with:
A. bitemporal hemianopsia.
B. cataracts.
C. glaucoma.
D. homonymous hemianopsia.
B19 C2 II
Following a traumatic brain injury, a patient is inconsistently oriented to time and place. She is unable to remember recent
events and shows little or no carry-over for new learning. The PRIMARY goal of rehabilitation at this stage of recovery is
to promote:
A. environmental and daily structure with consistency of cognitive stimuli.
B. increased arousal and attention through the use of sensory stimulation techniques.
C. independence in problem solving skills in varying environments.
D. varying daily tasks and structure to increase ease of cognitive retrieval.
B20 C2 II
A patient incurred a right CVA one month ago. She demonstrates moderate tone in her left upper extremity (predominantly
flexor tone). Her major problem at this time is a lack of voluntary movement control and her left upper extremity shows
minimal active movement with 1/4-inch subluxation of the shoulder. An appropriate initial activity would be:
A. PNF D2 flexion pattern, left upper extremity.
B. quadruped, rocking from side to side.
C. sitting, left active shoulder protraction with extended arm.
D. sitting, weight bearing on extended left arm, weight shifting.
333687839.doc

Practice Questions

B21 C7 III
A therapist conducts a study of the effectiveness of hot and cold in treating patients with pain. He recruits 200 patients for
each treatment group. The pain instrument he uses has a possible total score of 50, with 50 being the worst pain. His data
analysis reveals that Group A (heat modalities) has a mean score of 33 with a standard deviation of 1.0 while Group B (cold
modalities) had a mean of 35 with a standard deviation of 6.0. Based on this data the conclusion one should reach is:
A. cold has a greater effect on pain relief than heat.
B. heat has a greater effect on pain relief than cold.
C. the spread of scores with cold treatment demonstrates variability is greater.
D. the spread of scores with heat treatment demonstrates variability is greater.
B22 C1 I
The therapist in the photograph is testing which muscle?

A.
B.
C.
D.

anterior deltoid.
middle deltoid.
supraspinatus.
upper trapezius.

B23 C6 II
A patient presents with pain and instability of the left foot/ankle secondary to poliomyelitis with more recent development
of progressive post-polio muscle atrophy. In this case, a plastic solid ankle-foot orthosis is an appropriate prescription in
order to:
A. control excessive amounts of knee flexion during swing.
B. maintain dorsiflexion throughout swing.
C. provide modest assistance to dorsiflexion while restricting plantar flexion.
D. restrict all movement.
B24 C1 I
A patient is referred to physical therapy following an anterior dislocation of the right shoulder. A possible positive
examination finding as the result of this trauma would be:
A. positive drop arm test.
B. positive Neer test.
C. weak deltoids.
D. weak rhomboids.
B25 C8 I
A 3 month-old infant is being examined at an early intervention program. The therapist is having difficulty flexing the right
upper extremity to remove the infants clothing. This task is made more difficult if:
A. the infant is turned into sidelying on the left.
B. the infant sits up with support of the head in a neutral position.
C. the infants head is turned to the left.
D. the infants head is turned to the right.

333687839.doc

Practice Questions

B26 C7 III
A therapist has been asked to give an inservice presentation to staff nurses on safe guarding techniques. In order to best
prepare for this talk, the therapist should:
A. provide a questionnaire to a random sampling of participants one week before the scheduled presentation.
B. provide a questionnaire to all participants two weeks before the scheduled session.
C. survey the audience a day before the scheduled session.
D. survey the audience at the scheduled session.
B27 C2 III
The patient with left hemiplegia would be least likely to respond in therapy if the motor learning strategies emphasized:
A. encouragement of the patient to slow down.
B. maximum use of demonstration and gesture.
C. maximum use of verbal cues.
D. simplification/restructuring of the environment including removal of all clutter.
B28 C3 II
A two-week old infant born at 27 weeks gestation with hyaline membrane disease is referred for a physical therapy consult.
Nursing reports that the child "desaturates to 84% with handling" and has minimal secretions at present. The physical
therapist should:
A. delegate to a physical therapist assistant for a maintenance program of manual techniques for secretion clearance.
B. perform manual techniques for secretion clearance, 2 to 4 hours daily, to maintain airway patency.
C. provide suggestions to nursing for positioning for optimal motor development.
D. put the PT consult on hold as the child is too ill to tolerate exercise.
B29 C1 II
A patient sustained a valgus stress to his left knee while skiing. His orthopedist found a positive McMurrays test, and a
positive Lachmans stress test. He has been sent to physical therapy for conservative management of his dysfunction. The
sub-acute phase of physical therapy intervention should emphasize:
A. closed-chain functional strengthening of the quadriceps femoris and hamstrings emphasizing regaining terminal
knee extension.
B. closed-chain functional strengthening of the quadriceps femoris and hip abductors to promote regaining terminal
knee extension.
C. open-chain exercises of the hip extensors and hamstrings to inhibit anterior translation of the femur on the tibia.
D. open-chain strengthening of the quadriceps femoris and hip adductors to inhibit anterior translation of the tibia on
the femur.
B30 C8 I
The most enthusiastic visual tracking by a neonate would be elicited by:
A. a black and white face, with a red nose, held approximately nine inches from the infants eyes, moved horizontally.
B. a multicolored clowns face, held 12 inches away and moved horizontally.
C. a multicolored spinning top placed 15 inches from the infants eyes.
D. a red ring on a string, held six inches away from the infants eyes and moved horizontally.
B31 C2 II
A 14 year-old patient with traumatic brain injury has a convulsive seizure during a therapy session. She lost consciousness
and presents with tonic-clonic convulsions of all extremities. The therapists BEST response is to:
A. initiate rescue breathing immediately and call for help to restrain her.
B. position in sidelying, check for an open airway, and immediately call for emergency assistance.
C. position in supine with head supported with a pillow and wait out the seizure.
D. wrap her limbs with a sheet so she cant hurt herself and position in supine.
B32 C7 II
A therapist wants to know whether NDT handling techniques produces an improvement in independent rolling that lasts
longer than 30 minutes. In this study rolling is the:
A. control variable.
B. dependent variable.
C. independent variable.
D. intervening variable.
B33 C3 I
333687839.doc

Practice Questions

A computer programmer, with no significant past medical history, presents to the emergency room with complaints of fever,
shaking chills and a worsening productive cough. He has chest pains over the posterior base of his left thorax which is
made worse on inspiration. Which of the following is NOT an appropriate physical finding for this patient?
A. asymmetrical breathing.
B. crackles over the posterior aspect of his left thorax.
C. limited chest excursion.
D. slowed respiratory rate.
B34 C6 II
A patient is demonstrating genu valgum during standing and walking. This problem can be effectively controlled by
prescribing a knee-ankle-foot orthosis with:
A. anterior knee cap strap.
B. posterior plastic shell.
C. pretibial and suprapatellar anterior bands.
D. quadrilateral brim.
B35 C4 I
A 62 year-old patient is confined to bed following a severe stroke, She has 25 year history of diabetes and diabetic
neuropathy in both lower extremities. During an examination of sensory and integumentary integrity, the therapist identifies
an ischemic necrosis on the lateral side of the right foot in the region of the fifth metatarsal head. Hypoesthesia is found in
both feet and she is not able to sense a 5.07 Semmes-Weinstein monofilament in the right forefoot. The therapists
interpretation is that:
A. protective sensation of the foot is lost at the 10 g force level.
B. the foot is insensate with protective sensation loss at the 75 g force level.
C. the patient in unable to perceive sharp/dull sensation.
D. the patient still has normal protective sensation in the right foot.
B36 C5 III
A patient strained his lower back muscles three weeks ago and now complains of pain (6/10). Upon examination, the
therapist identifies bilateral muscle spasm from T10-L4. The therapist elects to apply interferential current to help reduce
pain and spasm. The MOST APPROPRIATE electrode configuration to choose in this case would be:
A. four electrodes with current flow diagonal to the spinal column.
B. four electrodes with current flow perpendicular to the spinal column.
C. two electrodes with current flow parallel to the spinal column.
D. two electrodes with current flow perpendicular to the spinal column.
B37 C8 I
An infant has just begun to pull-to-stand through kneeling, still demonstrates plantar grasp in standing, and is independent
in sitting including all protective extension reactions. This infants chronological age is approximately:
A. 5-6 months.
B. 7-8 months.
C. 9-10 months.
D. 12-14 months.
B38 C1 II
A sixty-five year old male retired bus driver has an increasing frequency of low back pain over the last 10 years. He states
that NSAIDs help to relieve his symptoms but there is always a nagging type pain. He reports significant stiffness in the
morning which dissipates by noon after exercising and walking his dog. Pain is exacerbated with frequent lifting and
bending activities as well as sitting for long periods. Physical therapy intervention should emphasize:
A. joint mobilization, soft tissue mobilization, and flexion exercises.
B. modalities to reduce pain, joint mobilization, and lumbar extension exercises.
C. modalities to reduce pain, postural reeducation and dynamic stabilization exercises.
D. postural reeducation, soft tissue mobilization, and dynamic stabilization.

333687839.doc

Practice Questions

B39 C7 III
A therapist is orienting a new physical therapy aide in transfer techniques. Initial consideration would be to:
A. ask about previous work and other experiences related to transferring individuals.
B. determine specific goals for teaching the techniques.
C. give a computer simulated instructional program before actual hands on training takes place.
D. provide an organized series of talks dealing with patient safety during transfers.
B40 C7 II
An elderly patient with diabetes and bilateral lower extremity amputation is to be discharged from an acute care hospital
two weeks post surgery. The incisions on his residual limbs are not healed and continue to drain. He is unable to transfer
because the venous graft sites in his upper extremities are painful and not fully healed. His endurance out-of-bed is limited.
The MOST appropriate discharge destination for this patient would be:
A. custodial care facility.
B. home.
C. rehabilitation hospital.
D. skilled nursing facility.
B41 C2 I
A patient currently being seen for low back pain awoke one morning with drooping left facial muscles and excessive
drooling. He was recovering from a cold and had experienced an earache in his left ear during the previous 2 days. The
therapist suspects a Bells Palsy which can be confirmed by examining:
A. corneal reflex and stretch reflexes of facial muscles.
B. taste over the anterior tongue and having the patient raise his eyebrows and puff cheeks.
C. taste over the posterior tongue and having the patient protrude the tongue.
D. trigger points for pain, especially over the TMJ.
B42 C6 I
A physical therapist assistant is assigned to ambulate a 72 year-old patient with Parkinsons disease. The therapist instructs
the PTA to watch for:
A. an abnormally wide base of support.
B. decreased trunk rotation with shorter steps.
C. unsteady, uneven gait with staggering steps to one side.
D. wider strides and increased double support time.
B43 C3 I
A patient with congestive heart failure is on a regimen of diuretics and calcium channel blockers. The potential side-effects
of these medications that the physical therapist should be alert for include:
A. decreased electrolytes and electrical instability evidenced by increased arrhythmias.
B. gastrointestinal upset and extreme fatigue.
C. orthostatic hypotension and dizziness.
D. reflex tachycardia and unstable blood pressure.
B44 C1 II
A patient with a traumatic injury to her right hand had a flexor tendon repair to the fingers. Physical therapy intervention
following this type of repair would begin:
A. within a few days following surgery to preserve tendon gliding.
B. within a few days following surgery to allow for early initiation of strengthening exercises.
C. after the splint is removed in 2-3 weeks to allow full active range of motion of all affected joints.
D. after the splint is removed in 4-6 weeks to allow ample healing time for the repaired tendon.
B45 C8 I
An 89 year-old community dwelling elder fell in her home and suffered multiple fractures of her right arm including Colles
fracture of her right wrist, and humeral fracture of her right shoulder. She is hospitalized for open reduction, internal
fixation of her right radius. Which is NOT an expected finding with this patient?
A. decreased pain and tenderness at the fracture sites.
B. increased likelihood of developing complications like pneumonia.
C. mental confusion following hospitalization and surgery.
D. slower healing time with prolonged rehabilitation.

333687839.doc

Practice Questions

B46 C2 I
The therapist suspects a patient recovering from a middle cerebral artery stroke is exhibiting a pure hemianopsia. This can
be examined using a:
A. distance acuity chart placed on a well-lighted wall at patients eye level 20 feet away
B. penlight held 6 inches from the eyes and moved inward toward the face.
C. penlight held approximately 12 inches from the eyes and moved to the extremes of gaze right and left.
D. visual confrontation test with a moving finger.
B47 C6 I
A patient is recovering from a left tibial amputation and complains of numbness and tingling affecting his dorsal foot and
big toe. This is driving him crazy because he knows the limb is gone. The therapist suspects the source of his discomfort is
most likely pressure from residual limb wrapping affecting the:
A. common peroneal nerve.
B. medial calcaneal nerve.
C. sural nerve.
D. tibial nerve.
B48 C7 II
A therapist investigated the accuracy of pulse oximeter estimates during exercise. Correlational analysis measured the
strength of the relationship between two types of ear-probe-equipped pulse oximeters during heavy cycle exercise under
hypoxic conditions. The investigator found measured arterial oxyhemoglobin saturation (%HbO2) levels to have a
correlation of .89 at high saturation but only .68 at low saturation levels. The results of this study suggest:
A. accuracy of the measurements increases at higher saturation levels.
B. both machines are highly accurate at all saturation levels.
C. both machines are only moderately accurate.
D. during heavy exercise, oxygen saturation levels should be interpreted cautiously.
B49 C1 I
A college soccer player sustained a hyperextension knee injury when kicking the ball. The patient was taken to the
emergency room of a local hospital and was diagnosed with "knee sprain". He was sent to physical therapy the next day for
aggressive rehabilitation. As part of the examination to determine the type of treatment plan to implement, the therapist
conducted the test shown in the figure. Based on the test picture, the therapist is examining the integrity of the:

A.
B.
C.
D.

anterior cruciate ligament.


iliotibial band.
medial meniscus.
posterior cruciate ligament.

B50 C1 II
A patient is seen in physical therapy 2 days post motor vehicle accident. The chief complaints are headaches, dizziness,
neck pain with guarding, and a "sensation of a lump in the throat". Plain film x-rays were read as negative. The therapist
should refer this patient for:
A. a second series of plain film x-rays
B. CT scan.
C. myelogram.
D. T-2 MRI.

333687839.doc

Practice Questions

B51 C5 II
A patient with an acutely sprained ankle will be treated by immersion into an ice water bath. The therapist should tell the
patient to expect in order:
A. aching, numbness, and burning, followed by intense cold.
B. burning, intense cold, and aching, followed by numbness.
C. intense cold, burning, and aching, followed by numbness.
D. numbness, aching, and intense cold, followed by burning.
B52 C8 III
A 94 year-old patient with cancer is admitted to a local hospital, She is given an Advanced Care Medical Directive to
review and sign. Guidelines for this document include specifying the rights of the individual patient to:
A. be informed of a facilitys right to modify the format and implementation schedule of the document.
B. be informed of the right to make decisions regarding their medical care upon admission.
C. establish mental competence for decision making, with signing witnessed by two adults.
D. insist on contraindicated palliative treatment even if the ultimate outcome is death.
B53 C2 II
A patient is recovering from stroke. At 4 months he is ambulating with a straight cane for household distances. In the clinic,
when his cane is taken away and he practices ambulating with no assistive device, recurvatum is observed that worsens
with continued walking. The therapists BEST strategy is to:
A. give him a KAFO to control the hyperextension and have him use a hemi walker.
B. give him a small based quad cane to improve his stability and have him practice AROM in supine.
C. practice isolated small range quadriceps eccentric control work in standing and continue with the straight cane.
D. put him on a Cybex and work on increasing quadriceps torque output at higher loads and increasing speeds.
B54 C6 II
A patient recovering from an incomplete spinal cord injury at the L3 level (ASIA Scale D) ambulates with bilateral
Lofstrand crutches. He remarks that he has great difficulty going down ramps and reports his knees wobble and are
unsteady. An appropriate intervention for this problem would be:
A. biofeedback training to reduce a knee extensor spasticity.
B. progressive resistance training for the quadriceps.
C. prolonged icing to reduce hamstring pain.
D. stretching using a posterior resting splint for tight plantar flexors.
B55 C7 III
It is illegal to release patient information without obtaining the patients consent to:
A. another healthcare provider involved in the care of the patient.
B. the insurance company that is paying for the patients treatment.
C. the patients employer when the condition is work-related.
D. the referring physician when there is a referral relationship.
B56 C6 III
With respect to the workers sitting postures in the Figure, the greatest reduction in lumbar spine compression forces would
be achieved by:

A.
B.
C.
D.

decreasing the chair backrest-seat angle to 85 degrees.


eliminating armrests on the chair.
increasing the angle between the seat pan and backrest to between 90-110 degrees.
using a 2 inch gel seat cushion.
333687839.doc

10

Practice Questions

B57 C1 I
A baseball pitcher has been sent to physical therapy with progressive posterior shoulder pain and weakness of the shoulder
abductors and lateral rotators. The therapist notices muscle wasting superior and inferior to the scapular spine. The patients
problem is MOST LIKELY attributable to damage involving the:
A. long head of the biceps brachii.
B. scalene muscles.
C. spinal accessory nerve.
D. suprascapular nerve.
B58 C7 III
Nursing homes that receive Medicare reimbursement for eligible residents are required by law to provide for rehabilitation
services including physical therapy based on:
A. diagnostic categories (DRGs).
B. needs assessment performed by a physical therapist.
C. referral from a physician.
D. referral from the nurse case manager.
B59 C2 II
A patient is recovering from surgical resection of an acoustic neuroma. She presents with symptoms of dizziness, vertigo,
horizontal nystagmus, and postural instability. To address these problems, her physical therapy plan of care should
incorporate:
A. Hallpike exercises to improve speed in movement transitions.
B. prolonged bed rest to allow vestibular recovery to occur
C. repetition of movements and positions that provoke dizziness and vertigo.
D. strengthening exercises focusing on spinal extensors.
B60 C6 I
A patient is beginning her ambulation training with a right above-knee prosthesis. The therapist notices that during early
swing the heel rises excessively. A possible cause is:
A. amputee pain.
B. too little tension in the extension aid.
C. too much knee friction.
D. too much tension in the extension aid.
B61 C4 II
A therapist is treating a 93 year-old patient in the home environment. Upon arrival, the therapist notices that the patient is
confused and skin color and turgor are poor. The patient reports of an intestinal bug for the last few days with frequent
vomiting and diarrhea. The therapists BEST course of action is to:
A. cancel therapy for today, carefully document the findings and notify the physician.
B. give the patient water and notify the physician immediately.
C. monitor vital signs, if HR is not elevated, get the patient up and walking.
D. notify the family, and insist that the patient should not be alone until the illness is over.
B62 C3 II
A patient who is 5 weeks post-myocardial infarction is participating in a cardiac rehabilitation program. The therapist is
monitoring responses to increasing exercise intensity. The indicator that exercise should be immediately terminated is:
A. 1.5 mm of downsloping ST segment depression.
B. appearance of a PVC on the ECG.
C. peak exercise HR greater than 140.
D. systolic blood pressure greater than 140 mm Hg or diastolic BP greater than 80 mm Hg.
B63 C8 II
An infant who was 33 weeks gestational age at birth and is now 3 weeks chronological age demonstrates colic. In this case,
the BEST intervention the physical therapist could teach the mother is:
A. fast vestibular stimulation.
B. neutral warmth.
C. stroking and tapping.
D. visual stimulation with a colored object.

333687839.doc

Practice Questions

11

B64 C2 I
A therapist is treating a patient with Brown-Sequard syndrome that resulted from a gun shot wound. The therapists
examination should reveal:
A. loss of motor function and pain and temperature sensation with preservation of light touch and position sense
below the level of the lesion.
B. loss of upper extremity function, (cervical tract involvement) with preservation of lower extremity function
(lumbosacral tract involvement).
C. sparing of tracts to sacral segments with preservation of perianal sensation, and active toe flexion.
D. ipsilateral weakness and loss of position sense and vibration below the lesion level with contralateral loss of pain
& temperature sensation.
B65 C7 II
A patient is recovering from stroke and presents with moderate impairments of the left upper and lower extremities. The
therapists goal today is to instruct him in a stand-pivot transfer to his affected side so he can go home on a weekend pass.
His wife is attending todays session and will be assisting him on the weekend. The BEST choice for teaching this task is
to:
A. demonstrate the task, then have the caregiver practice with the patient.
B. demonstrate the task, then practice with the patient.
C. practice the task first with the caregiver, then with the patient.
D. practice the task first with the patient, then with the caregiver.
B66 C7 III
A 78 year-old currently in an acute care facility recovering from fractures of the spine and hip. His Medicare Part A benefits
have come to an end and he is soon to be discharged. Medicare Part B will NOT cover:
A. durable medical equipment needed for the patient in his home.
B. home health agency rehabilitation services.
C. outpatient therapy services in a private therapy practice.
D. outpatient therapy services in an acute care facility.
B67 C6 I
When using a patellar-tendon bearing prosthesis, a patient will experience excessive knee flexion in early stance if the:
A. foot position is inset too much.
B. foot position is outset too much.
C. socket is aligned too far back or tilted posteriorly.
D. socket is aligned too far forward or tilted anteriorly.
B68 C3 II
A chronic smoker developed carcinoma of the lung. Following a right pneumonectomy, post-operative physical therapy
intervention should include:
A. breathing exercises to both sides of the thorax to maintain adequate aeration.
B. percussion in all postural drainage positions.
C. shaking in all postural drainage positions.
D. sustained maximal inspiration training with an incentive spirometer.
B69 C6 II
A physical therapist has to order a wheelchair for a patient with a T9-10 spinal cord injury who wishes to engage in sports
activities. The wheelchair WILL NOT need:
A. a high back.
B. a low back.
C. removable arm rests.
D. swing-away footrests.
B70 C1 II
A patient complains of persistent wrist pain after painting her house three weeks ago. The patient demonstrates signs and
symptoms consistent with deQuervains tenosynovitis. An appropriate special test which can be used to confirm the
diagnosis is:
A. Craigs test.
B. Finkelsteins test.
C. Froments sign.
D. Phalens test.
333687839.doc

12

Practice Questions

B71 C8 II
An adaptive wheelchair for a child with moderate spastic quadriplegic cerebral palsy would include:
A. a pommel to keep hips abducted.
B. movable chairback to allow for extension of the hips and trunk.
C. movable footrests to allow ankles to plantarflex.
D. pads on the seat to keep hips adducted.
B72 C2 I
A 73 year-old with persistent balance difficulty and a history of recent falls (2 in the last 3 months) is referred for physical
therapy examination and evaluation. During the initial session it is crucial to examine:
A. cardiovascular endurance during a 6 minute walking test.
B. level of dyspnea during functional transfers.
C. sensory losses and sensory organization of balance.
D. spinal musculoskeletal changes secondary to degenerative joint disease.
B73 C7 III
A patient with adhesive capsulitis of the shoulder sustains a fracture of the shoulder during treatment provided by the PTA.
The fracture occurred while the PTA was mobilizing the shoulder joint, which was part of the plan of care established by
the PT. After the incident, the PTA told the PT that she was not familiar with the mobilization techniques to the shoulder.
Responsibility in this case falls on:
A. both the PT and the PTA are responsible for establishing effective communication regarding the skills and
competencies of the PTA.
B. neither the PT nor the PTA are responsible for the fracture, it is an accepted risk associated with joint mobilization.
C. the PT who is solely responsible for assessing the competence of the PTA under their supervision.
D. the PTA who is responsible for informing their supervising PT whenever they are unfamiliar or uncomfortable
with any treatment procedure.
B74 C7 III
A patient is referred for physical therapy with a diagnosis of second degree ankle sprain. The therapist is busy writing a
discharge summary when the patient with the ankle sprain arrives for his scheduled appointment. The physical therapist
assistant on staff knows the patient as he had treated the patient previously for a similar injury. The physical therapist
should:
A. ask the physical therapist assistant to commence the examination.
B. ask the physical therapist assistant to examine the patients PROM to speed up the process.
C. complete the written discharge summary at another time and examine the patient.
D. tell the patient to wait until the following day for a complete examination.
B75 C1 I
During a postural screen for a patient complaining of low back pain the therapist notices that the knees are in genu
recurvatum. Possible contributory postures would include:
A. ankle dorsiflexion and hip abduction.
B. ankle plantarflexion and anterior pelvic tilt.
C. forefoot varus and posterior pelvic tilt.
D. lateral tibial torsion and anterior pelvic tilt.
B76 C1 I
After treating a patient for trochanteric bursitis for one week, the patient has no resolution of pain and is complaining of
problems with gait. After re-examination the therapist finds weakness of the quadriceps femoris, and altered sensation at the
greater trochanter. This is most likely due to:
A. degenerative joint disease of the hip.
B. L4 nerve root compression.
C. L5 nerve root compression.
D. sacroiliac dysfunction.

333687839.doc

Practice Questions

13

B77 C4 I
A patient in chronic renal failure is being seen in physical therapy for deconditioning and decreased gait endurance. The
therapist needs to schedule the patients sessions around dialysis which is received 3 mornings a week. The patient is also
hypertensive and requires careful monitoring. The therapists BEST approach is to take blood pressure:
A. every minute during activity, using the shunt arm.
B. in the seated position when activity has ceased.
C. in the supine position, using the shunt arm.
D. pre and post activities, using the non-shunt arm.
B78 C6 II
A patient is recovering from a fracture of both tibia and fibula in the right lower leg which has been casted. The referral is
for gait training, nonweightbearing on right lower extremity. The safest gait pattern for this patient to use is a:
A. four-point.
B. swing-to.
C. three-point.
D. two-point.
B79 C7 II
A therapist wishes to determine the effectiveness of TENS on the relief of pain in a group of 20 patients with phantom limb
pain. She recruits her patients over a 2 year period. All receive a 6 week intervention. She finds that 12 patients with
phantom pain got pain relief while 8 had no relief. The BEST conclusion that can be reached is that this:
A. Level 1 randomized controlled trial provides conclusive evidence of the effectiveness of TENS.
B. Level 2 cohort design allows small but definitive conclusions to be reached.
C. Level 3 case-control study provides limited confidence in the effectiveness of TENS.
D. Level 4, a quasi-experimental study, provides only questionable evidence of treatment effectiveness.
B80 C3 II
Following a post-myocardial infarction, a patient is a new admission to a Phase 3 cardiac rehabilitation program. During the
initial exercise session his ECG responses are continuously monitored via radio telemetry. The therapist notices three PVCs
occurring in a run with no P wave. The MOST appropriate action to take is to:
A. continue the exercise session but monitor closely.
B. have him sit down and rest for a few minutes before resuming exercise.
C. modify the exercise prescription by decreasing the intensity.
D. stop the exercise and notify the physician immediately.
B81 C2 I
A patient with left hemiplegia is able to recognize his wife after she is with him for awhile and talks to him, but he is unable
to recognize the faces of his children when they come to visit. The children are naturally very upset by their fathers
A. anosognosia.
B. ideational apraxia.
C. somatognosia.
D. visual agnosia.
B82 C8 I
A home care therapist receives a referral to evaluate the fall risk potential of an 82 year-old community dwelling elder. He
has fallen three times in the last four months with no history of fall injury except for minor bruising. He is currently taking
a number of medications. The drug LEAST likely to contribute to his increased fall risk is:
A. Buffered aspirin.
B. Digoxin.
C. Elavil.
D. Nitroglycerin.
B83 C1 II
A patient complains of foot pain when first arising which eases with ambulation. The therapist finds that symptoms can be
reproduced in weightbearing and running on a treadmill. Examination reveals pes planus and pain with palpation at the
distal aspect of the calcaneus. Early management would include:
A. modalities to reduce pain
B. prescription for a customized orthosis.
C. strengthening of ankle dorsiflexors.
D. use of a resting splint at night.
333687839.doc

14

Practice Questions

B84 C6 III
A patient with a complete T10 paraplegia resulting from a spinal cord injury is ready to begin community wheelchair
training. The therapists goal is to teach him how to do a wheelie so he can manage curbs. The BEST training strategy is to
instruct the patient to:
A. grasp the handrims posteriorly and pull them forward abruptly and forcefully.
B. lean backward while moving the hands slowly backward on the rims.
C. place his hand on the top of the handrims to steady the chair while he throws his head and trunk forward.
D. throw his head and trunk backward to rise up on the large wheels.
B85 C3 I
A physical therapist should be alert to recognize the signs and symptoms associated with the onset of aspiration pneumonia.
Of the following, the patient MOST susceptible to develop this form of pneumonia is one with:
A. a circumferential burn of the thorax associated with significant pain.
B. a complete spinal cord lesion at T2 with diminished coughing ability.
C. amyotrophic lateral sclerosis with dysphagia and diminished gag reflex.
D. severe scoliosis with compression of internal organs.
B86 C6 II
A patient demonstrates quadriceps weakness (4-/5) and difficulty descending stairs. The BEST intervention to regain
functional strength in the quadriceps is:
A. isokinetic exercise, at 36 degrees/second.
B. maximum isometric exercise, at 45 and 90 degrees of knee extension.
C. partial squats progressing to lunges.
D. progressive resistance exercises, 70% 1 RM, 3 sets of 10.
B87 C2 I
Independent community ambulation as the primary means of functional mobility is a realistic functional expectation for a
patient with the highest level of spinal cord injury at:
A. high lumbar (T12-L1).
B. low lumbar (L4-L5).
C. low thoracic (T9-10).
D. midthoracic (T6-T9).
B88 C5 II
A patient presents with multiple fractures of both hands and wrists as a result of a mountain bike accident. Now five weeks
later, the patient currently has vertigo, limited wrist and finger motion, and dry scaly skin over the involved areas. The
physical agent MOST appropriate to select in this case would be:
A. contact ultrasound.
B. functional electrical stimulation.
C. hot packs.
D. paraffin.
B89 C6 II
A patient is 82 years-old and demonstrates a history of recent falls (two in the last two months) and mild balance instability.
The therapists referral is to examine the patient and recommend an assistive device as needed. Based on the patients
history, it would be BEST to select a:
A. a folding reciprocal walker.
B. front wheel rolling walker that folds.
C. hemi walker.
D. standard, fixed frame walker.
B90 C1 III
Following surgery, a patient develops a stiff pelvis and limited pelvic/lower trunk mobility. The therapist elects to use
sitting exercises on a therapy ball to correct these impairments. In order to improve lower abdominal control, the ball would
have to move:
A. backward, producing anterior tilting of the pelvis.
B. backward, producing posterior tilting of the pelvis.
C. forward, producing anterior tilting of the pelvis.
D. forward, producing posterior tilting of the pelvis.
333687839.doc

Practice Questions

15

B91 C2 I
A patient presents with weakness in his right lower leg 3 weeks following a motor vehicle accident. He complains of
spontaneous twitching in the muscles of the lower leg. The therapist visually inspects both limbs and determines that
muscle bulk is reduced on the involved right limb. Girth measurements confirm a 1 inch difference in the circumference of
the right leg measured 4 inches below the patella. Deep tendon reflexes and tone are diminished. Based on these signs and
symptoms the therapist concludes the patient is exhibiting:
A. a peripheral nerve injury.
B. brainstem dysfunction affecting extrapyramidal pathways.
C. Guillain-Barr syndrome.
D. pyramidal track dysfunction above the decussation in the medulla.
B92 C2 I
A patient with syringomyelia was found on physical examination to have impairment of pain and temperature sensation of
the face but preservation of light touch. This dissociated sensory loss in the face can be explained by involvement of:
A. the main sensory nucleus of CN V.
B. the mesencephalic nucleus of CN V.
C. the nucleus of the tractus solitarius of CN VII.
D. the spinal nucleus of CN V.
B93 C3 II
A patient with bacterial pneumonia has crackles and wheezes at her left lateral basal segment and decreased breath sounds
throughout. She in on 4 liters of oxygen by nasal cannula which brings her SaO2 to 90%. Respiratory Rate is 28. The
intervention that would be INAPPROPRIATE to use in this case is:
A. breathing exercises encouraging expansion of the left lateral basilar thorax.
B. percussion to the appropriate area on the left lateral basilar thorax.
C. shaking over the appropriate area on the left lateral basilar thorax.
D. standard postural drainage for the lateral basal segment, left lower lobe.
B94 C8 I
A therapist receives a referral to see an elderly patient in the ICU recovering from a severe case of pneumonia. The
therapist recognizes his disorientation is due to delirium rather than dementia because:
A. he demonstrates persistent personality changes.
B. he has hallucinations throughout the day.
C. his level of arousal is significantly depressed.
D. his symptoms are intermittent.
B95 C7 II
A comparison of the effects of exercise in water, on land, or combined on the rehabilitation outcome of patients with intraarticular anterior cruciate ligament reconstructions revealed that less joint effusion was noted after 8 weeks in the water
group. An appropriate statistical test to compare the girth measurements of the three groups is:
A. analysis of covariance.
B. analysis of variance.
C. chi square.
D. Spearman rho.
B96 C2 II
A patient has a recent history of strokes (2 in the past 4 months). He demonstrates good return of his right lower extremity
and is walking with a straight cane. The therapist is concentrating on improving his balance and independence in gait.
Unfortunately his speech recovery is lagging behind his motor recovery. He demonstrates a severe fluent aphasia. The
BEST strategy to use during physical therapy is:
A. consult with the speech pathologist to establish a communication board.
B. demonstrate and gesture to get the idea of the task across.
C. have the family present to help interpret during PT sessions.
D. utilize verbal cues, emphasizing consistency and repetition.

333687839.doc

16

Practice Questions

B97 C6 I
An individual is walking with an above-knee prosthesis and demonstrates terminal swing impact. The therapist suspects:
A. insufficient knee friction.
B. the hip flexors are weak.
C. the prosthesis is externally rotated.
D. too little tension in the extension aid.
B98 C4 II
A patient was burned over 40% of his body in an industrial accident. He has full thickness burns over his anterior trunk and
neck and superficial partial thickness burns over his shoulders. The MOST appropriate procedures to stabilize this patient
out of positions of common deformity include:
A. a cervical thoracic lumbo-sacral orthosis (CTLSO) used during all upright activities.
B. plastic cervical orthosis and axillary splints utilizing an airplane position.
C. soft cervical collar with an intrinsic plus hand splint.
D. splints utilizing a flexed position for the shoulders and body jacket for the trunk.
B99 C1 I
A 16 year-old patient, seen in the emergency room, was placed in a short leg cast following a severe right-sided Grade III
ankle sprain. She is immediately referred to physical therapy for gait training using crutches. She complains that her right
great toe is numb and she cannot lift her toes. This problem MOST LIKELY is a result of:
A. anxiety and hyperventilation in anticipation of using crutches.
B. compression of the common peroneal nerve.
C. compression of the medial plantar nerve.
D. compression of the posterior tibial artery resulting in vascular insufficiency.
B100 C2 III
Which intervention would be LEAST likely used to improve left-sided neglect in a patient with left hemiplegia?
A. bridging with both arms positioned in extension at the sides.
B. hooklying, lower trunk rotation, and lightly resisted "knee rocks".
C. rolling, supine to sidelying on right, using a PNF lift pattern.
D. sitting, with hands forward resting on large ball, weight shifting moving ball to the left.
B101 C7 III
A patient with traumatic brain injury has been receiving in-patient physical therapy for the past two months has not
demonstrated meaningful improvement in functional status. The patients care is covered by Medicare. The therapist has
informed both the physician and the family of his lack of progress. The family insists that the therapist continue to treat the
patient, and the physician will continues to certify the patient for more physical therapy. The therapist should:
A. continue to provide the care both the family and the referring physician demand; it is the physicians
responsibility to determine the appropriateness of physical therapy.
B. modify treatment goals in a manner that will allow the therapist to demonstrate that the treatment is achieving
progress towards reasonable goals.
C. provide the family Medicare "notification of noncoverage" information, and carefully explain it, and their options,
which could include paying for the care out-of-pocket.
D. refer the patient to another clinic that is willing to continue treatment despite lack of functional improvement.
B102 C3 II
A patient with COPD reports to his fourth outpatient pulmonary rehabilitation session complaining of nausea, gastric upset
and feeling jittery. The patient reports no change in pulmonary symptoms. The physical therapist records the following set
of vital signs: temperature 98.6F, heart rate 110 beats/min, and irregular blood pressure 150/86, respiratory rate 20. Breath
sounds show no change from baseline. The therapist checks the medical record and finds that the patient has no history of
gastric disease. He is presently taking theophylline, Ventolin, and Amcort. The physical therapist should:
A. call the patients physician and report signs of theophylline toxicity.
B. have the patient increase his use of Ventolin to improve respiratory status.
C. have the patient stop his use of Amcort until he can schedule an appointment with his physician.
D. send the patient home and have him notify the physician of his current symptoms.

333687839.doc

Practice Questions

17

B103 C8 I
The therapist is treating a one year-old child with Down syndrome at home and notices decreasing strength in the
extremities with neck pain and limited neck motion. DTRs are 1+. The therapists suspects:
A. atlantoaxial subluxation with lemniscal impingement.
B. atlantoaxial subluxation with spinal cord impingement.
C. lower motor neuron signs consistent with Down syndrome.
D. upper motor neuron signs consistent with Down syndrome.
B104 C2 I
Which of the following causative factors is NOT likely to produce shoulder pain in hemiplegia?
A. flaccid paralysis with loss of seating action of rotator cuff muscles.
B. passive range of motion while emphasizing scapulohumeral rhythm.
C. spastic retraction with depression and downward rotation of scapula.
D. traction and gravitational forces acting on a depressed, downwardly rotated scapula.
B105 C6 I
While gait training a patient recovering from a cerebral vascular accident, the therapist observes the knee on the affected
side going into recurvatum during stance phase. The MOST LIKELY cause of this deviation can be attributed to:
A. severe spasticity of the hamstrings or weakness of the gastrocnemius-soleus.
B. weakness of both the gastrocnemius-soleus and pretibial muscles.
C. weakness of the gastrocnemius-soleus or spasticity of the pretibial muscles.
D. weakness or severe spasticity of the quadriceps.
B106 C4 II
Your examination of the pressure ulcer pictured on the patients heel reveals dry eschar without edema, erythema,
fluctuance or drainage. The patient is afebrile. The BEST recommendation regarding intervention for this ulcer is:
A. enzymatic debridement.
B. need for an arterial by-pass graft.
C. sharp debridement.
D. use of an AFO with heel pressure relief.
B107 C7 II
A therapist has completed a study investigating the relationship between ratings of perceived exertion (RPE) and type of
testing modality: arm ergometry versus leg ergometry. The therapist finds a correlation 0.59 with the arm testing while the
correlation is 0.79 with the leg testing. Interpretation of these results is that:
A. both arm and leg ergometry are highly correlated with RPE.
B. both arm and leg ergometry are only moderately correlated with RPE.
C. leg ergometry is highly correlated with RPE while arm ergometry is only moderately correlated.
D. the common variance of both types of testing is only 22%.
B108 C1 II
A patient has a diagnosis of left knee degenerative joint disease. He complains of left-sided knee pain of two months
duration. He has been followed by outpatient physical therapy for three weeks. The patient feels his condition is worsening
as pain has increased during weight bearing activities and he can no longer fully extend his left knee. Examination findings
include: increased swelling, decreased knee AROM into extension, and an antalgic gait. The physical therapist should:
A. continue physical therapy for another two weeks because there is uncertainty if the patient understands or is
complying with the home exercise program.
B. continue therapy for another week to ensure that all interventions have been attempted and then return the patient
to the referring physician.
C. immediately return the patient to the referring physician with documentation indicating that treatment was
ineffective.
D. tell the patient to see an orthopedic surgeon for possible immediate surgical intervention.

333687839.doc

18

Practice Questions

B109 C5 III
Two months ago, a patient had an inversion sprain affecting the calcaneofibular and anterior talofibular ligaments of the
right ankle. The ankle is still painful, very limited in motion and slightly tender to the touch. As part of PT intervention,
ultrasound treatment parameters should consist of:
A. continuous US @ 1 MHz.
B. continuous US @ 3 MHz.
C. pulsed US @ 1 MHz.
D. pulsed US @ 3 MHz.
B110 C2 III
In Neurodevelopmental Treatment (NDT) of the patient recovering from stroke, therapy would NOT include:
A. facilitation of early movement in synergistic patterns followed quickly by movement training in out-of-synergy
patterns.
B. facilitation of selective movement control in out-of-synergy patterns.
C. functional activities emphasizing bimanual tasks and reintegration of the hemiplegic side.
D. reduction of spasticity and abnormal reflex activity through positioning and handling techniques.
B111 C8 II
A 4 year-old child with moderate spastic diplegia is referred to physical therapy for an adaptive equipment evaluation.
Which apparatus would be inappropriate to recommend?
A. bilateral KAFOs.
B. posture control walker (posterior walker).
C. prone stander
D. tone reducing AFOs.
B112 C6 II
A patient is recovering from a left CVA. He is wearing a plastic knee-ankle-foot orthosis to stabilize his right foot. During
gait analysis the therapist observes lateral trunk bending toward the right as he bears weight on the right leg at midstance.
The BEST choice to correct this problem is:
A. provide a lift on the shoe of the involved leg.
B. strengthen hamstrings on the right side.
C. strengthen hip flexors on the right side.
D. strengthen the hip abductors on the right side.
B113 C1 II
A ten year-old male who plays catcher on his baseball team complains of bilateral knee pain which is exacerbated with
forceful quadriceps contraction. He has also noticed pain and swelling at the distal attachment of the patellar tendon.
Effective early physical therapy intervention should include:
A. AROM exercises to prevent contracture.
B. casting followed by decreased loading of the knee.
C. decreased loading of the knee by the quadriceps femoris muscle.
D. modalities to decrease inflammation.
B114 C3 II
An 83 year-old patient has a decubitus ulcer of 3 months duration on his lateral ankle. The ankle is swollen, red, and painful
with a moderate to high amount of wound drainage (exudate). The BEST dressing for this wound is:
A. calcium alginate dressings.
B. gauze dressings.
C. hydrogel dressings.
D. semipermeable film dressings.
B115 C1 I
A patient was diagnosed with a bulging disc at the right L5-S1 spinal level without nerve root compression. The
manifestation of this problem MOST LIKELY to be documented is:
A. centralized gnawing pain with loss of postural control during lifting activities.
B. centralized gnawing pain with uncompensated gluteus medius gait.
C. radicular pain to the right great toe with a compensated gluteus medius gait.
D. radicular pain to the right great toe with difficulty sitting for long periods.

333687839.doc

Practice Questions

19

B116 C2 II
High level training for an individual recovering from traumatic brain injury who demonstrates Rancho Cognitive Function
Level VII should focus on:
A. involving the patient in decision making and monitoring for safety.
B. providing a high degree of environmental structure to ensure correct performance.
C. providing assistance as needed using guided movements during training.
D. providing maximum supervision as needed to ensure successful performance and safety.
B117 C6 II
To correct for the problem of a forward festinating gait in a patient with Parkinsons disease, the therapist could:
A. increase cadence using a metronome.
B. increase stride length using floor markers.
C. use a heel wedge.
D. use a toe wedge.
B118 C6 III
A patient presents with difficulty with fast movement speeds and fatigues easily. The therapist decides on a strength training
program that specifically focuses on improving fast-twitch fiber function. The optimal exercise prescription to achieve this
goal is:
A. high intensity workloads for long durations.
B. high intensity workloads for short durations.
C. low intensity workloads for long durations.
D. low intensity workloads for short durations.
B119 C2 I
The therapist is called to the traumatic brain injury unit to examine a new patient. Among the findings, are ptosis and miosis
of the right eye with loss of facial sweating. The therapist suspects:
A. loss of parasympathetic function of the oculomotor nerve.
B. sympathetic nerve damage consistent with Argyll-Robertson pupil.
C. sympathetic nerve damage consistent with autonomic dysreflexia.
D. sympathetic nerve damage consistent with Horners syndrome.
B120 C8 II
A physical therapy functional goal for a 5 year-old child with a very high lumbar lesion myelomeningocele (L2 level) and
minimal cognitive involvement would be:
A. community ambulation with a reciprocating gait orthosis (RGO) and Lofstrand crutches.
B. community ambulation with HKAFOs and Lofstrand crutches.
C. household ambulation with a reciprocating gait orthosis (RGO) and Lofstrand crutches.
D. household ambulation with KAFOs and rollator walker.
B121 C7 II
A patient with multiple sclerosis is part of a national study testing the effectiveness of a new medication. The patient reports
that the pill she is taking makes her feel much better and allows her to move easier. At the conclusion of the study it is
revealed that she was part of the control group. Her responses are MOST likely due to:
A. Hawthorne effect.
B. placebo effect.
C. pretest-treatment interference.
D. sampling bias.
B122 C3 I
The therapist is reading a recent report of arterial blood gas analysis with the following values: Fraction of inspired oxygen
(FiO2) - 0.21, PaO2 - 53 mmHg, PaCO2 - 30 mmHg, pH - 7.48, Bicarbonate ion - 24 mEq/l This would indicate that the
patient is in:
A. metabolic acidosis.
B. metabolic alkalosis.
C. respiratory acidosis.
D. respiratory alkalosis.

333687839.doc

20

Practice Questions

B123 C4 III
A 32 year-old patient is referred to a cardiac exercise group following a mild myocardial infarction. From her intake
questionnaire the therapist learns she has Type I (IDDM) diabetes, controlled with twice daily insulin injections. In order to
minimize the risk of a hypoglycemic event during exercise, the therapist should have the patient:
A. avoid exercise during periods of peak insulin activity.
B. exercise daily for 40-50 minutes to achieve proper glucose control.
C. have the patient decrease her carbohydrate intake for 2 hours before the exercise session.
D. monitor blood glucose levels carefully every week during the rehabilitation program.
B124 C8 II
An 85 year-old resident of a community nursing home is diagnosed with Alzheimers type dementia. In formulating a plan
of care, it is important to understand that the patient:
A. can usually be trusted to be responsible for her daily care needs.
B. can usually be trusted with transfers with appropriate positioning of the wheelchair.
C. is more likely to remember current experiences then past ones.
D. will likely be resistant to activity training if unfamiliar activities are used.
B125 C2 III
A patient is having difficulty learning how to transfer from mat to wheelchair. The patient just cant seem to get the idea of
how to coordinate this movement. In this case, the MOST effective use of feedback during early motor learning is:
A. focus on guided movement and proprioceptive inputs.
B. focus on knowledge of performance and proprioceptive inputs.
C. focus on knowledge of results and visual inputs.
D. provide feedback only after a brief (5 second) delay.
B126 C6 III
A patient presents with pain and paresthesias over the first two metatarsal heads of her right foot. Pain is worse after
prolonged periods of weight bearing. She typically wears shoes with 3 inch heels and pointed toes. The BEST intervention
is a:
A. pad placed distal to the metatarsal heads.
B. pad placed proximal to the metatarsal heads.
C. scaphoid pad to support the medial longitudinal arch.
D. Thomas heel to support the medial longitudinal arch.
B127 C1 II
A patient developed right throbbing shoulder pain after painting the kitchen. Passive and active glenohumeral motions
increase pain. The BEST initial intervention for this acute shoulder condition is:
A. correction of muscle imbalances to allow healing of right shoulder supraspinatus tendinitis.
B. modalities to reduce pain and inflammation as the result of subdeltoid bursitis.
C. rotator cuff strengthening exercises to allow ADL function following biceps tendinitis.
D. stretching of the pectoralis minor muscle following acromioclavicular joint inflammation.
B128 C4 I
A patient has a 5 year history of AIDS. The case worker reports a gradual increase in difficulty with walking. The patient
rarely goes out anymore. A referral to P.T. is initiated. Examination findings reveal typical neuromuscular changes
associated with AIDS. These deficits would likely include:
A. motor ataxia and paresis with pronounced gait disturbances.
B. paraplegia or tetraplegia.
C. progressive rigidity and akinesia with severe balance disturbances.
D. widespread sensory loss resulting in sensory ataxia.
B129 C7 III
A new graduate physical therapist has an appointment for a job interview with Human Resources at a large teaching
hospital. She is well dressed, has a professional typewritten resume, and is prompt for her 2:00 p.m. appointment. The
interviewer should AVOID discussing:
A. benefits of working in a large teaching hospital.
B. health insurance and vacation policy.
C. the applicants marital status.
D. work hours and sick time.
333687839.doc

Practice Questions

21

B130 C8 III
A therapist is treating a two year-old child with Down syndrome who frequently uses a W sitting position. The main reason
to discourage W sitting in this child is that it may cause:
A. abnormally low tone because of reflex activity.
B. developmental delay of normal sitting.
C. femoral antetorsion and medial knee stress.
D. hip subluxation and lateral knee stress.
B131 C2 I
A 92 year-old patient is being examined for signs of stroke. When tested for two-point discrimination on his right hand, he
is unable to tell whether the therapist is touching him with one or two points. The therapist determines that there is impaired
function in the:
A. anterior spinothalamic tract or thalamus.
B. dorsal column/lemniscal pathways or somatosensory cortex.
C. lateral spinothalamic tract or somatosensory cortex.
D. spinal lemniscus or ventral posterolateral nucleus of the thalamus.
B132 C3 II
A 75 year-old patient has a history of two myocardial infarctions and one episode of recent congestive heart failure. He also
has claudication pain in the right calf during his exercise tolerance test. An INITIAL exercise prescription that BEST deals
with his problems is walking:
A. daily, using interval training for 10 to 15 minute periods.
B. five times a week using continuous training, for 60 minutes.
C. three times a week using continuous training, for 40 minute sessions.
D. three times a week using interval training, for 30 minute periods.
B133 C7 III
A valid informed consent for research purposes should include a number of elements. The one that is NOT essential is:
A. a statement ensuring the subjects commitment to participate for the duration of the study.
B. all potential benefits of participation.
C. all reasonable and foreseeable risks and discomforts.
D. an understandable explanation of the purpose and procedures to be used.
B134 C4 II
A patient demonstrates post-partum sacral pain. The patient complains pain is increased with prolonged walking, ascending
or descending stairs, and rising from sit-to-stand. Upon examination, the therapist finds a laterally tilted pelvic with
restricted mobility. A standing ipsilateral flexion kinetic test reveals movement of the PSIS cranially with hip hiking. The
intervention that is NOT appropriate for this problem is:
A. cryotherapy and TENS for relief of pain and muscle spasm.
B. increasing non-weightbearing and stabilization using a lumbosacral orthosis.
C. increasing weight bearing tolerance with the temporary use of a sacroiliac belt.
D. mobilization of the sacroiliac joint to restore alignment and mobility.
B135 C5 III
A patient has severe low back pain as a result of chopping wood and has been receiving ultrasound and strengthening
exercises to the low back for two weeks. The therapist has also been applying TENS and decides to have the patient use
TENS at home. As a safety precaution, it is important to instruct the patient to perform a daily check of the:
A. electrodes and electrode jacks.
B. electrodes and leads.
C. skin and electrodes.
D. skin and leads.
B136 C6 III
A tilt-in-space wheelchair has been ordered for a patient with C4 quadriplegia. This type of chair is ordered to:
A. facilitate handgrip and propulsion.
B. improve leg position and lower extremity edema.
C. improve positioning for pressure relief or extensor spasms.
D. improve the patients ability to relieve pressure and transfer independently.

333687839.doc

22

Practice Questions

B137 C8 II
An 82 year-old female has low vision. She recently returned home from a 2 week hospitalization for stabilization of her
diabetes. The therapists goal is to mobilize her and increase her ambulation level and safety. The BEST intervention
strategy for this patient is to:
A. practice walking in areas of high illumination and low clutter.
B. keep window shades wide open to let in as much light as possible.
C. practice walking by having her look at her feet at all times.
D. color code stairs with pastel shades of blue and green to highlight steps.
B138 C3 I
The patient has a history of angina pectoris and limited physical activity. As the patient participates in the second exercise
class the therapist suspects that angina is unstable and may be indicative of a preinfarction state. The therapist determines
this by the presence of:
A. angina of increasing intensity that is unresponsive to the nitroglycerin or rest.
B. angina that responds to rest and interval training but not to continuous training
C. arrhythmias of increasing frequency, especially atrial arrhythmias.
D. prolonged cessation of pain following the administration of nitroglycerin for angina.
B139 C2 III
A patient with a left CVA has left him hemiparetic on the right side. In his current stage of recovery, he demonstrates strong
and dominant hemiplegic synergies in the leg. Which activity would NOT be helpful to break up these synergies?
A. foot tapping in a sitting position.
B. weight shifts in kneeling.
C. assuming the bridging position.
D. rolling from the hooklying position using lower extremity D1 flexion PNF pattern.
B140 C7 III
Criteria have been established by Medicare that designate a patient as homebound and allow for home physical therapy
services to be approved. The criterion that does NOT apply in determining homebound status is:
A. dependency on others for all transportation needs.
B. inability to safely leave home unattended.
C. ambulation for short distances causing dyspnea and chest pain.
D. leaving the home three times a week to receive dialysis.
B141 C6 I
A patient with an above-knee prosthesis is walking by swinging the prosthesis out to the side in an arc during swing of the
amputated limb. The therapist suspects the prosthesis may have:
A. a stiff knee mechanism.
B. an excessively low lateral wall.
C. an unstable knee unit.
D. insufficient support from the anterior wall.
B142 C4 II
An 87 year-old patient has been hospitalized for three weeks following a surgical resection of carcinoma of the colon. He is
very weak and the therapist is instructing him in ambulation with a walker. He complains of pain in his left shoulder which
is aggravated by weight bearing when using the walker. The therapist decides to:
A. notify the physician immediately.
B. apply pulsed ultrasound to decrease pain.
C. apply heat in the form of a hot pack prior to ambulation.
D. ambulate the patient in the parallel bars considering age and diagnosis.
B143 C5 II
A college athlete sprained the left ankle 4 days ago. The patient complains of pain (4/10), and there is moderate swelling
which is getting worse. At this time, which intervention would be BEST to use?
A. cold/intermittent compression combination with the limb elevated.
B. intermittent compression followed by elevation.
C. contrast baths followed by limb elevation.
D. cold whirlpool, followed by limb elevation.

333687839.doc

Practice Questions

23

B144 C1 I
A patient diagnosed with left lateral epicondylitis has no resolution of symptoms after two weeks of treatment. The therapist
begins a re-examination and finds the left biceps reflex is 1+. The therapist should NEXT perform a complete examination
of the:
A. mid-cervical region.
B. cervical-thoracic region.
C. upper cervical region.
D. cervico-cranial region.
B145 C3 II
An obese patient who is 70 pounds overweight is recovering from a mild myocardial infarction and needs cardiovascular
conditioning. The exercise class will be used in conjunction with a dietary program to promote weight reduction. The
MOST appropriate exercise prescription for this patient is:
A. walking, intensity set at 50% target heart rate.
B. jogging, for 10 minutes at 4 miles per hour.
C. walking, intensity set at 75% of heart rate reserve.
D. swimming, intensity set at 75% age-adjusted heart rate.
B146 C1 I
A patient underwent a total hip replacement (THR) four months ago. The patient is now referred to physical therapy for gait
evaluation. The patient demonstrates shortened stride length on the right. This patient MOST LIKELY has:
A. contracted hip flexors.
B. contracted hamstrings.
C. weakened hip flexors.
D. weakened quadriceps.
B147 C8 III
A child with spastic cerebral palsy is having difficulty releasing food from her hand into her mouth. Once the child has
brought the food to the mouth it would be helpful for the caregiver to:
A. give a quick stretch to the finger extensors.
B. passively extend the fingers.
C. slowly stroke the finger extensors in a distal to proximal direction.
D. slowly stroke the finger flexors in a distal to proximal direction.
B148 C2 II
A patient with amyotrophic lateral sclerosis presents with early signs of progressive muscle weakness and muscle
cramping. He also has minimal spasticity in his lower extremities. The therapists INITIAL exercise prescription should
focus on:
A. active exercises since resistive exercises are contraindicated.
B. low intensity general conditioning exercises.
C. maximizing exercise and activity.
D. PROM exercise to maintain joint flexibility since active exercise is contraindicated.
B149 C6 II
In reference to the figure, when lifting a constant load using either a stoop lift or squat lift posture, the most significant
contributing factor for increasing the lumbar spine compression forces is:

A.
B.
C.
D.

performing the lift with the lumbar spine in a kyphotic posture using a stoop lift technique.
performing the lift with the lumbar spine in a neutral position rather than a lordotic posture.
the distance of the load from the base of the spine.
the height of the load from the ground.
333687839.doc

24

Practice Questions

B150 C1 II
Nearly two months ago, a patient noticed that she had left shoulder pain after walking her dog. This pain has progressively
worsened. She now is unable to move her left upper extremity overhead while performing activities of daily living. An
orthopedic surgeon diagnosed her problem as adhesive capsulitis. The MOST effective direction for glenohumeral
mobilization for this patient would be:
A. anterior inferior translatory glides.
B. anterior superior translatory glides.
C. posterior inferior translatory glides.
D. posterior superior translatory glides.
B151 C8 I
A 79 year-old patient is referred to physical therapy for an examination of his functional mobility skills and safety in the
home environment. The family reports the patient is demonstrating increasing forgetfulness and some memory deficits.
From the examination, the therapist would expect to find:
A. impairments in short-term memory.
B. periods of agitation and wandering, especially in the late afternoon.
C. periods of fluctuating confusion.
D. significant impairments in long-term memory.
B152 C2 I
A therapist is examining a patient with vestibular dysfunction. The patient is asked to assume a long sitting position with
the head turned slightly to one side. The therapist then quickly moves the patient backward so that the head is extended
over the end of the table approximately 30 below horizontal. The maneuver causes dizziness and vertigo. The therapist
reports these findings as:
A. a negative positional test.
B. a positive Clinical Test for Sensory Interaction in Balance (CTSIB).
C. a positive Hallpike maneuver.
D. a positive sharpened Romberg test.
B153 C7 III
A therapist is instructing a patient with a transfemoral lower extremity amputation in prosthetic gait training. The therapist
determines that learning is going well because the patients errors are decreasing and overall endurance is improving. The
BEST strategy to promote continued motor learning at this point in the patients rehabilitation is to:
A. have the patient continue to practice in the parallel bars until all errors are extinguished.
B. have the patient practice walking in varying environments.
C. intervene early whenever errors appear before bad habits become firmly entrenched.
D. provide continuous feedback after every walking trial.
B154 C3 III
All of the following would be appropriate home physical therapy interventions for a young patient with cystic fibrosis
except:
A. autogenic drainage for secretion removal once or twice daily
B. functional activities to increase endurance 3 to 5 times per week.
C. secretion removal techniques by the parents to all lobes once or twice a day.
D. using the a capella device in postural drainage positions one or twice a day
B155 C4 II
A 55 year-old is referred to a womans health clinic with problems of stress incontinence. She reports loss of control that
began with coughing or laughing but now reports she has problems even when she exercises (she does aerobics 3 times a
week). The BEST intervention for this patient is:
A. behavioral modification techniques to reward proper voiding on schedule.
B. biofeedback one hour a week to achieve appropriate sphincter control.
C. functional electrical stimulation three times a week.
D. Kegel exercises several times a day.

333687839.doc

Practice Questions

25

B156 C2 I
A 76 year-old patient suffered a cerebral thrombosis four days ago. She presents with the following symptoms: decreased
pain and temperature sensation of the ipsilateral face; nystagmus, vertigo, and nausea; dysphagia, ipsilateral Horners
syndrome, and contralateral loss of pain and temperature sensation of the body. The MOST LIKELY site of the thrombosis
is the:
A. internal carotid artery.
B. midbasilar artery.
C. posterior cerebral artery.
D. posterior inferior cerebellar artery.
B157 C5 II
A patient with a two inch stage II decubitus ulcer over the left lateral malleolus is referred for physical therapy. The
therapist notes a greenish, pungent exudate at the wound site. The therapist decides to use electrical stimulation. The BEST
choice of polarity and electrode placement is:
A. anode placed in the wound.
B. anode placed proximal to wound.
C. cathode placed in the wound.
D. cathode placed proximal to wound.
B158 C3 III
Running on a level gym surface is an appropriate Phase 2 outpatient cardiac rehabilitation activity for some patients
because the MET level is of:
A. high intensity (15 METs) and useful for high functioning patients.
B. low intensity, (below 5 METs) with little skill variability between individuals expected.
C. moderate intensity, (13 METs) with moderate skill variability between individuals.
D. moderate intensity, (8-9 METS) and can be easily alternated with walking at 5 METs.
B159 C4 II
A 27 year-old woman presents with a complete T10 paraplegia. An extensive neurological work-up has failed to reveal a
specific cause for her paraplegia. Her physician has determined that she has a conversion disorder. During physical therapy
it would be BEST to:
A. focus on the underlying causes for the paralysis in an empathetic manner.
B. initiate functional training consistent with her level of injury.
C. initiate ROM and strength training while she receives psychological counseling.
D. use functional electrical stimulation as a means of demonstrating to her that the muscles function.
B160 C7 III
A certified cardiopulmonary clinical specialist, was moved off the coronary care unit to treat a patient who had a surgical
repair of a lacerated index finger flexor tendon. The therapist had dealt almost exclusively with patients with cardiovascular
disease; however, the department was very short-staffed. During the treatment the patient felt a pop which was the result
of a rupture of the newly repaired tendon. The physical therapist in this case should have:
A. refused to treat this patient.
B. requested more time to read the literature about this repair.
C. treated the patient as requested.
D. used heat prior to the treatment.
B161 C8 I
Which is NOT considered a normal finding during an examination of a newborn infant?
A. continuous tremulousness.
B. dramatic skin color changes with change of state.
C. response decrement to repetitive stimuli.
D. symmetry in range of motion.
B162 C6 I
A patient comes into an outpatient clinic using a wheelchair with excessive leg length from the seat to the foot plate. The
therapist suspects this is the cause of his:
A. excessive forward leaning.
B. excessive weight bearing on the ischial seat.
C. sacral sitting and sliding forward in the chair.
D. uneven thigh weight distribution.
333687839.doc

26

Practice Questions

B163 C4 I
A patient presents with a large sacral decubitus ulcer that is purulent and draining. The therapist needs to take a
representative sample of the infected material in order to obtain a laboratory culture. The MOST appropriate method to
culture this wound is to obtain samples from the:
A. dressing and exudate in the wound.
B. dressing, exudate, and surrounding bed linen.
C. exudate in the wound and the surrounding tissues.
D. exudate in the wound.
B164 C3 III
A Phase 2 outpatient cardiac rehabilitation program uses circuit training with different exercise stations for the 50 minute
program. One station uses arm ergometry. For arm exercise as compared to leg exercise, at a given workload, the therapist
can expect:
A. both HR and systolic/diastolic BP will be higher.
B. exercise capacity is reduced due to higher stroke volumes.
C. HR will be higher while systolic BP will be lower.
D. the principal change is higher systolic and diastolic BP.
B165 C7 II
A therapist wants to determine whether a treatment was effective in reducing lower extremity edema in a group of patients
with peripheral vascular edema. Volumetric measurements using a water displacement method is selected as the outcome
measure. The data was compared to a control group receiving no treatment. Analysis of this data is BEST done by
employing:
A. A. chi square.
B. B. ANOVA.
C. Pearsons product moment.
D. t-test.
B166 C8 II
In planning a physical activity program for an elderly patient with Alzheimers disease, it is MOST important that the daily
activities:
A. are changed daily to meet the need for variety.
B. are highly structured to reduce anxiety and confusion.
C. promote involvement and interest through maximum stimulation.
D. provide maximum opportunities for making choices.
B167 C2 III
A patient has a complete spinal cord injury at the level of C6. The therapist is instructing the family in exercises to maintain
passive range of motion. The therapist wants the family to focus on:
A. keeping all muscles fully ranged through normal ROM.
B. keeping muscles fully ranged, with hyperflexibility in the low back extensors and hamstrings
C. limiting range of motion in the shoulders to promote stability.
D. ranging individual muscles according to specific functional needs.
B168 C7 III
A physical therapist and physical therapist assistant are conducting a cardiac rehabilitation class for twenty patients. The
therapist is suddenly called out of the room. The MOST appropriate procedure in this situation is to:
A. have the patients switch to less intense exercise until the therapist returns.
B. have the PTA supervise the class in cool-down activities.
C. have the PTA supervise the class using the outlined exercise protocol until the therapist returns.
D. terminate the exercises and have the patients monitor their pulses until the therapist returns.
B169 C1 III
The most efficient intervention to regain biceps brachii strength if the muscle is chronically inflamed and has a painful arc
of motion is:
A. active concentric contractions through partial range of motion.
B. active eccentric contractions in the painfree range.
C. isokinetic exercises through the full range of motion.
D. isometric exercises at the end range of movement only.
333687839.doc

Practice Questions

27

B170 C5 III
During an ultrasound treatment the patient flinches and states that a strong ache was felt in the treatment area. To address
this patients concern, it would be BEST to:
A. add more transmission medium.
B. decrease the ultrasound frequency.
C. decrease the ultrasound intensity.
D. increase the size of the treatment area.
B171 C7 III
The Back to Work Center, which specializes in work conditioning, is scheduled for an accrediting site survey. The
appropriate agency to conduct this program is the:
A. Commission on Accreditation of Rehabilitation Facilities (CARF).
B. Department of Health & Human Services.
C. Joint Commission on Accreditation of Health Care Organizations (JCAHO).
D. Occupational Safety & Health Administration (OSHA).
B172 C4 II
A patient is recovering from deep partial thickness burns over the posterior thigh and calf which are now healed. The
therapists examination reveals local tenderness with swelling and pain on movement in the hip area. While palpating the
tissues the therapist detects a mass. The therapists BEST course of action is to:
A. continue with ROM exercises but proceed gently.
B. report these findings promptly to the physician.
C. use petrissage to work on this area of focal tenderness.
D. use RICE to quiet down the inflammatory response.
B173 C8 I
A 10 month-old (corrected age) infant born at 23 weeks gestational age suffered Grade III intraventricular hemorrhage
prenatally and was on a ventilator for 2 months after birth. During a physical therapy examination, increased resistance to
passive movement is noted in all extremities, but most markedly in the lower extremities. The infant will probably be
diagnosed with:
A. ataxic cerebral palsy.
B. spastic diplegic cerebral palsy.
C. spastic hemiplegic cerebral palsy.
D. spastic quadriplegic cerebral palsy.
B174 C3 I
ECG changes that occur with exercise in an individual with myocardial ischemia and coronary artery disease would be
expected to include:
A. bradycardia with ST segment depression greater than 3 mm below baseline.
B. significant arrhythmias early on in exercise with a shortened QRS.
C. significant tachycardia early on in exercise with ST segment elevation.
D. tachycardia at a relatively low intensity of exercise with ST segment depression.
B175 C1 II
A 45 year-old female patient with long term postural changes exhibits an excessive forward head and complains of pain and
dizziness when looking upward. The MOST effective physical therapy intervention is:
A. anterior cervical muscle stretching and postural reeducation to relieve vertebral artery compression.
B. posterior cervical muscle stretching and postural reeducation to relieve vertebral artery compression.
C. upper cervical Grade III mobilization to reduce pressure on the cervical spinal nerves.
D. upper cervical joint mobilization Grade II and postural reeducation to reduce compression of the cervical
sympathetic ganglia.
B176 C6 I
A patient has an above-knee prosthesis with a prosthetic foot that includes a plantar flexion bumper. During ambulation the
therapist decides that the bumper is too stiff. This decision is based on the presence of:
A. foot slap at midstance.
B. high heel rise at toe-off.
C. lateral foot rotation at heel strike.
D. swing phase whip.
333687839.doc

28

Practice Questions

B177 C3 III
During the course of the physical therapy treatment in the ICU, a radial artery line gets pulled (comes out of the artery). The
FIRST thing the physical therapist should do is:
A. elevate the arm above heart level to stop the bleeding.
B. place a blood pressure cuff on the involved extremity and inflate the cuff until the bleeding stops.
C. push the code button in the patients room as this is a cardiac emergency.
D. re-insert the arterial catheter into the radial artery and check the monitor for an accurate tracing.
B178 C2 I
A patient suffers a CVA resulting in right hemisphere damage. This patient will MOST LIKELY exhibit:
A. hesitancy, requiring more feedback and support.
B. negative, self-deprecating comments and frequent depression.
C. poor judgment with increased safety issues.
D. slow, cautious behaviors.
B179 C4 II
A patient with a methicillin-resistant staphylococcal infection (MRSA) has been discharged from an isolation setting as an
inpatient and is now returning to physical therapy as an outpatient. The precaution that needs to be adhered to is that:
A. direct contact with the patient should be avoided.
B. open wounds must be contained within a dressing.
C. treatment should be performed in an isolated private room.
D. whirlpool for wound care should be avoided.
B180 C1 I
After examining a patient who was referred to physical therapy for posterior thoracic pain, the therapist finds no
musculoskeletal causes for the patients symptoms. Pain may be referred to this thoracic region from the:
A. appendix.
B. gall bladder.
C. heart.
D. ovary.
B181 C6 III
An 82 year-old patient with a transfemoral amputation is being fitted with a temporary prosthesis containing a SACH
prosthetic foot. This prosthetic foot:
A. absorbs energy through a series of bumpers, permitting sagittal plane motion only.
B. allows full sagittal and frontal plane motion.
C. allows limited sagittal plane motion with a small amount of mediolateral motion.
D. is an articulated foot with multiplanar motion.
B182 C8 II
An 82 year-old patient demonstrates significant proprioceptive losses in both lower extremities, distal greater than
proximal. The BEST strategy to assist in compensatory gait training is to have him:
A. count out loud during each step.
B. look at his feet for placement while walking.
C. practice walking on smooth tile floors.
D. use light touch down support on available furniture.
B183 C3 I
A patient is being seen for shortness of breath. Notable on physical examination is a deviated trachea to the left. Which of
the following processes would account for such a finding?
A. left pleural effusion.
B. left pneumothorax.
C. right hemothorax.
D. right lung collapse.

333687839.doc

Practice Questions

29

B184 C5 II
A patient complains of pain (7/10) in the shoulder region secondary to acute subdeltoid bursitis. As part of the plan of care
during the acute phase, the therapist elects to use conventional TENS which will modulate the pain primarily through:
A. ascending inhibition.
B. descending inhibition.
C. gate control mechanisms.
D. stimulation of endorphins.
B185 C1 I
A construction worker describes a sudden onset of back pain while trying to lift a heavy barrel with another worker. He
describes his pain as constant, unremitting at an intensity of 10/10 over the last three days. He is unable to work, but can
drive himself to the clinic for treatment. Medication has not influenced his symptoms. He states he has never had any other
back-related symptoms in the past. MOST LIKELY involved in his symptomatology is:
A. discal dysfunction.
B. early degenerative osteoarthritis.
C. neoplastic disease.
D. secondary gain.
B186 C3 I
Following myocardial infarction, a patient was placed on medications which included a beta-adrenergic blocking agent.
When monitoring this patients response to exercise, the therapist expects this drug will cause heart rate to:
A. be low at rest and rise linearly as a function of increasing workload.
B. be low at rest and rise very little with exercise.
C. increase proportionally to changes in diastolic blood pressure.
D. increase proportionally to changes in systolic blood pressure.
B187 C7 III
A therapist is working in an outpatient clinic that has provider contracts with many types of insurance plans. A large
percentage of the clinics patients have insurance coverage that has a contract that pays a flat rate of reimbursement per
episode of care. The supervisor tells the therapist never to treat these patients for more than five visits. The therapist should:
A. ignore the supervisor and treat the patients at the frequency and duration appropriate for their condition.
B. keep track of all patients insured by this plan, and make certain they average less than or equal to five visits in
aggregate.
C. resign from the clinic and report the supervisor to the states licensing board.
D. speak to the supervisor about concerns that this policy conflicts with the Code of Ethics.
B188 C4 III
A patient has a 20 year history of diabetes. Changes include vascular insufficiency and diminished sensation of both feet
with poor healing of a superficial skin lesion. It is important that he understand the precautions and guidelines on foot care
for people with diabetes. Which of the following is NOT appropriate for a therapist to stress?
A. inspect the skin daily for inflammation, swelling, redness, blisters, or wounds.
B. use daily hot soaks and moisturize the skin.
C. wash the feet daily with mild soap, dry thoroughly, and hydrate with moisturizing lotion.
D. wear jogging shoes that allow adequate room and change shoes frequently.
B189 C1 I
After completing an examination of a patient with shoulder pain, the physical therapist concludes that the cause is
subscapularis tendinitis. This clinical finding supportive of this conclusion is:
A. pain provoked with active glenohumeral external rotation.
B. pain provoked with passive glenohumeral external rotation.
C. painful resisted shoulder adduction.
D. tenderness at the greater tubercle of the humerus.
B190 C1 II
A nineteen year-old male soccer player sustained a Grade II inversion ankle sprain two weeks ago. Intervention in the early
subacute phase of rehabilitation would most likely include:
A. closed-chain lower extremity strengthening and proprioceptive exercises.
B. functional soccer-related drills.
C. mobilization at the talocrural and subtalar joints.
D. plyometric-based exercise program.
333687839.doc

30

Practice Questions

B191 C5 II
A patient is referred for outpatient care following a tendon transfer of the extensor carpi radialis longus. The muscle
strength tests poor (2/5) in spite of previous intensive therapy. The therapist elects to apply biofeedback to assist in
progressively increasing active motor recruitment. Initially, the EMG biofeedback protocol should consist of:
A. high detection sensitivity with recording electrodes placed closely together.
B. high detection sensitivity with recording electrodes placed far apart.
C. low detection sensitivity with recording electrodes placed closely together.
D. low detection sensitivity with recording electrodes placed far apart.
B192 C8 I
A therapist receives a home care referral from the nurse case manager. An 82 year-old has lost functional independence
after the recent death of his wife. His past medical history includes stroke with minimal residual disability. Currently he no
longer goes out of his house and rarely even gets out of his chair anymore. During the initial session the therapist
determines that depression may be the cause of his increasing inactivity based on the presence of:
A. complaints of increasing dizziness and palpitations.
B. low scores on the Geriatric Depression Scale.
C. sleep apnea and weight gain.
D. weight loss and social withdrawal.
B193 C7 III
A 40 year-old has been laid off his construction job for the last two years. He has a wife who is a homemaker and seven
small children. They are currently on welfare. While raking leaves he has a stroke and is admitted to the hospital. In this
case, the third party payer that would provide assistance is:
A. Medicaid.
B. Medicare Part A.
C. Social Security.
D. Workers Compensation
B194 C3 I
A patient has been on bedrest for two days following revascularization surgery involving a triple coronary artery bypass
graft. During his therapy session the therapist notices increased tenderness in the patients calf. The therapist should
immediately examine for:
A. any swelling in the calf or ankle.
B. bradycardia.
C. Homans sign.
D. lowered body temperature.
B195 C1 I
A patient referred for TMJ dysfunction states that she has had three episodes of her jaw locking in an open position. The
MOST LIKELY cause of her problems is:
A. disc displacement.
B. entrapment of the retrodiscal lamina.
C. impingement of the temporomandibular ligament.
D. lateral pterygoid muscle spasm.
B196 C4 II
A 62 year-old patient is referred to physical therapy after a fall and ORIF for a fracture of her right wrist. During the initial
examination, the therapist observes that her skin and eyes have a yellowish hue. The therapists BEST course of action is to:
A. continue with the treatment, a yellowish hue is an expected finding 3-4 days post ORIF.
B. document the findings and consult with the surgeon immediately following treatment.
C. send a copy of the examination results to the referring surgeon emphasizing the skin hue.
D. treat her problem with whirlpool and massage and reevaluate skin color post treatment.

333687839.doc

Practice Questions

31

B197 C1 II
A 75 year-old inpatient received a cemented total hip replacement two days ago. The physical therapy plan of care should
have as its number one priority:
A. active range of motion exercises and early ambulation using a walker, nonweightbearing.
B. passive range of motion exercises and gait training using crutches, weightbearing to tolerance.
C. patient education regarding positions and movements to avoid.
D. proper technique for transferring to the toilet.
B198 C7 III
A patient is referred by an orthopedist with a diagnosis of impingement syndrome of the shoulder. The initial PT
examination reveals signs and symptoms that are not consistent with this diagnosis and are more consistent with thoracic
spine pain and dysfunction. The therapist treats the patient consistent with PT findings without communicating with the
referring physician. Months later the therapist is sued by the patients estate. The patient died of undiagnosed metastatic
lung cancer. The therapist is:
A. not legally licensed to diagnose metastatic cancer, therefore cannot be held responsible for the patients death.
B. not responsible for the incorrect diagnosis because treatment was appropriate for the PT findings.
C. responsible for communicating findings to the referring physician when the findings are inconsistent with a
referring physicians diagnosis.
D. responsible for making the diagnosis of possible cancer consistent with the PT examination of the patient.
B199 C5 III
A patient with Stage II primary lymphedema of the right lower extremity is referred for physical therapy. Examination
reveals increased limb girth with skin folds/flaps evident. An important component of lymphedema management is manual
lymphatic drainage. This procedure should include:
A. decongesting the distal portions of the limb first and working proximally.
B. decongesting the proximal portions of the limb first and working distally.
C. decongesting the trunk after the limb segments.
D. deep tissue friction massage for several minutes on fibrotic areas.
B200 C6 II
A patient with a 10 year history of multiple sclerosis demonstrates 3+ extensor tone in both lower extremities. The therapist
needs to order a wheelchair. It would be BEST to recommend a:
A. electric wheelchair with toe loops.
B. standard wheelchair with a 30 degree reclining back.
C. standard wheelchair with elevating leg rests.
D. tilt-in-space wheelchair with a pelvic belt.

333687839.doc

32

Practice Questions

B1 C8 II
Reaching for a multicolored object while in an unsupported, guarded sitting position.
The appropriate task would include the six month gross developmental level activity of working on unsupported sitting
(standing and supine are not appropriate choices). The use of a multicolored object is more appropriate than a bland and
white object for a three year cognitive level.
B2 C1 II
Change to grade II mobilizations to reduce the joint and soft tissue irritation.
Changing to grade II mobilizations will allow the soft tissue structures to more slowly adapt to mobilization. If pain
persists for over 24 hours, the soft tissue and joint irritation may progress. Grade IV mobilization techniques will cause
further irritation and not allow the soft tissues to adapt to the stretch. Self stretching is not as efficient as mobilization
techniques and would not be the optimum treatment strategy.
B3 C7 II
Use frequent reinforcements for all desired behaviors
Behavioral modification is best achieved through use of positive reinforcements for all desired behaviors. Negative
behaviors should be ignored. Self correction is not a form of behavior modification.
B4 C2 III
Isokinetic training using low resistance at fast movement speeds.
Dysmetria is a coordination problem in which the patient is unable to judge the distance or range of movement (overshoots
or undershoots a target). Adding resistance (weights, water resistance or manual resistance with PNF) can assist the patient
in slowing down the movement and achieving better control. Low resistance, fast speed isokinetic training is
contraindicated.
B5 C6 III
Halo orthosis
Maximum stabilization of the cervical spine can be achieved with the use of a halo orthosis. It contains a ring that is
attached to the skull by four screws, four uprights that connect the ring to thoracic bank or thoracic plastic jacket.
B6 C3 II
Administer supplemental O2
Oxygen should only be used with a physicians order. Since the patients SaO2 is above 88%, there is not a clear indication
for its use. Allowing the patient to recover from exercise through rest and breathing exercises, the patients oxygen level
should return to baseline (assuming that it fell from baseline to 90%). The oxygen levels and patient symptoms should be
noted in the medical record.
B7 C2 I
Electrical silence
Inserting an EMG needle into a normal muscle causes a burst of electrical activity (meridional activity). After which the
muscle produces no sound (electrical silence). Fibrillation potentials are spontaneous activity seen in relaxed denervated
muscle and polyphasic potentials are produced in the contracted muscle undergoing reorganization.
B8 C4 II
Passive ROM exercise
Resistive exercise is contraindicated in patients with significant bony metastases, osteoporosis, or low platelet count
(<20,000 to 25,000).
B9 C1 II
Stretching of scalenes and sternocleidomastoid muscles for early thoracic outlet syndrome symptoms.
Patients with emphysema typically present with hypertrophy of the accessory breathing muscles to include the scalenes
which can compress the neurovascular structures at the thoracic outlet.
B10 C8 II
Immediately administer orange juice for his developing hypoglycemia
Hypoglycemia, an abnormally blood glucose, results from too much insulin (insulin reaction). It requires accurate
assessment of symptoms and proper intervention. Have him sit down and give him an oral sugar (e.g., orange juice). Once
the patient is stabilized, the physician should be notified. Profuse sweating does not usually accompany orthostatic
hypotension.
333687839.doc

Practice Questions

33

B11 C2 II
Decrease the intensity and duration but maintain a frequency of 3 times/week.
Clinical manifestations of postpolio syndrome include myalgias, new weakness as well as atrophy, and excessive fatigue
with minimal activity. Nonexhaustive exercise and general body conditioning are indicated but should be submaximal in
intensity. The patient should not exercise to the point of fatigue and exhaustion. A change in the exercise prescription is
warranted.
B12 C7 III
Provide real-life examples that link learning to personal experience.
The student learns best using an intuitive or global learning style. The student processes information all at once and will
learn best if information is connected to personal experiences and is present in practical, real-life experiences. The other
choices are appropriate choices for an analytical/objective type of learner.
B13 C7 II
Daily
This patient is over 65 years old and covered by Medicare for his health care. The patient must receive therapy daily (at
least 5-7 days per week) for reimbursement in an in-patient acute care hospital setting.
B14 C3 I
Reduce airway resistance and exercise-induced bronchospasm
Sympathomimetics are a class of drugs that mimic the effects of stimulation of body organs and structures by the
sympathetic nervous system. Albuterol is an antiasthmatic medication used to promote bronchodilation. Its primary effects
are on Beta 2 receptors in the bronchiole smooth muscle. It may also have an effect on Beta 1 receptors, producing
cardiovascular adverse reactions (hypertension and tachycardia, chest pain, increased BP, sweating, and blanching of the
skin. Chronic use can lead to loss of bronchodilation and paradoxical bronchoconstriction.
B15 C6 III
Reduce effusion and assist venous return.
The pressure exerted by water on an immersed object is equal on all surfaces (Pascals law). As the depth of immersion
increases, so does hydrostatic pressure. Increased pressure limits effusion, assists venous return, and can induce
bradycardia. The other choices do not related directly to hydrostatic pressure. Buoyancy of water provides an environment
of relative weightlessness, and assists in joint unloading and active movement. Hydromechanics, movement of water
molecules, increases the resistance of water as peed of movement increases. Hot water immersion (> 35 0C can increase
cardiovascular demands at rest and with exercise.
B16 C6 II
Strengthen hip extensors through bridging
Backward trunk lean (gluteus maximus gait) is the result of a weak gluteus maximus. It will cause increased difficulty
going up stairs or ramps. Functional training exercises such as bridging are indicated.
B17 C4 II
Sulfamylon
Sulfamylon penetrates through eschar and provides antibacterial control. Silver nitrate and nitrofurazone are superficial
agents that attach surface organisms. Panafil is a keratolytic enzyme used for selective debridement.
B18 C8 I
Cataracts
Cataracts, a clouding of the lens, results in a gradual loss of vision; central vision is lost first, then peripheral. Glaucoma
produces the reverse symptoms: loss of peripheral vision is first, then central, progressing to total blindness. Hemianopsia
is a field defect in both eyes which often occurs following stoke. There was no mention of CVA in the question.
B19 C2 II
Environmental and daily structure with consistency of cognitive stimuli. The patient demonstrates recovery consistent with
a confused state (Rancho Los Amigos Levels of Cognitive Function V/VI). The main focus should be on providing
environmental and daily structure to reduce distractions and help the patient process stimuli. With memory deficits and
little carryover for new learning, functional independence is not a realistic goal at this stage of recovery. Working on
improving arousal and attention levels may be appropriate for patients functional at lower cognitive recovery levels, Level
I/II.

333687839.doc

Answers

33

B20 C2 II
Sitting, weight bearing on extended left arm, weight shifting
Sitting, weight bearing and rocking on an extended left arm will help to decrease the flexor tone. It also provides joint
compression (approximation) at the shoulder which will help maintain shoulder position and stimulate stabilizing muscles.
Quadruped is too strenuous for this patient at this time and the other two activities demand more voluntary control than this
patient currently demonstrates.
B21 C7 III
The spread of scores with cold treatment demonstrates variability is greater.
The spread of scores with cold treatment (standard deviation of 6.0) indicates that this treatment produces more variable
effects that the heat treatment. All other conclusions cannot be determined based on the data presented and the statistical
analysis used.
B22 C1 I
Supraspinatus
The muscle being tested is the supraspinatus. The empty-can position puts the supraspinatus muscle it its most effective
position for contraction. Weakness may be a result of inflammation, neuropathy of the suprascapular nerve or from a
tendon tear
B23 C6 II
Restrict all movement.
A solid ankle-foot orthosis is indicated with severe pain or instability; it allows no movement. The other choices do not
address these problems.
B24 C1 I
Weak deltoids
Due to the anatomical position of the axillary nerve, it can be damaged by an anterior-inferior dislocation at the
glenohumeral joint. A drop arm test evaluates the integrity of the rotator cuff. A Neer Test evaluates impingement of the
shoulder. The rhomboids are innervated by the dorsal scapular nerve. Anatomically, the dorsal scapular nerve is medial
and posterior to the shoulder joint.
B25 C8 I
The infants head is turned to the right
Turning the infants head to the right will probably elicit the ATNR with extension of the right upper extremity. This will
not facilitate removal of the clothing.
B26 C7 III
Provide a questionnaire to all participants two weeks before the scheduled session.
A questionnaire to all participants represents the best method of needs assessment in this situation. Leaving the survey to
the day of or day before the presentation does not allow for adequate advanced planning to meet the groups needs.
B27 C2 III
Maximum use of demonstration and gesture.
The patient with left hemiplegia typically demonstrates visuospatial perceptual deficits. Maximum use of demonstration
and gesture would be inappropriate to assist this patient in the relearning of motor tasks.
B28 C3 II
Provide suggestions to nursing for positioning for optimal motor development.
Excessive handling of a premature infant can cause oxygen desaturation. It is in the best interests of the infant to limit the
number of handlers. At present, there is little information provided that would necessitate the physical therapist or PTA to
be a direct caregiver to this child. In this instance, the physical therapists role should be to assist nursing in developing
positioning schedules, positions for feeding, infant stimulation activities, etc.
B29 C1 II
Closed-chain functional strengthening of the quadriceps femoris and hamstrings emphasizing regaining terminal knee
extension.
The evaluation is suggestive of an unhappy trial injury. Closed chain exercises are emphasized during the subacute phase
to enhance functional control of the muscles surrounding the knee. Terminal extension must be achieved during this stage
if normal function is to occur. All other choices do not promote terminal knee extension.
333687839.doc

34

Answers

B30 C8 I
A black and white face, with a red nose, held approximately nine inches from the infants eyes, moved horizontally.
A neonate sees black, white, and red most clearly with a fixed focal length of approximately nine inches.
B31 C2 II
Position in sidelying, check for an open airway, and immediately call for emergency assistance.
This is an emergency situation. In order to prevent aspiration, turn the head to the side or position in sidelying. Check to
see if the airway is open, wait for tonic-clonic activity to subside before initiating artificial ventilation if needed.
B32 C7 II
Dependent variable.
The dependent variable is the change of difference in behavior (in this example, rolling) that results from the intervention
(NDT handling techniques or independent variable).
B33 C3 I
Slowed respiratory rate
Crackles are typical finding with an infiltrate. Since the patient is having pain, thoracic expansion would likely be limited
and asymmetrical. With a lower than normal tidal volume, a respiratory rate would have to be elevated, not slowed, to
maintain an adequate minute ventilation (respiratory rate times tidal volume = minute ventilation).
B34 C6 II
Posterior plastic shell.
Genu valgum can be controlled in a knee-ankle-foot orthosis with a properly fitter posterior plastic shell. Older braces may
utilize a valgus correction strap which wraps around the medial knee and buckles around the lateral upright.
B35 C4 I
Protective sensation of the foot is lost at the 10 g force level.
The neuropathic limb is prone to the development of neuropathic ulcers. Examination is facilitated by the use of Semmes
Weinstein monofilaments. Three sizes of monofilaments are typically used: (1) the 4.17 monofilament provides 1 g of
force and is indicative of normal sensation, (2) the 5.07 monofilament provides 10 g of force and is indicative of protective
sensation loss, and (3) the 6.10 monofilament provides 75 g of force and reveals a severely insensate foot. Monofilaments
are used to examine pressure sensation (a single point perception test) and do not reveal any information about sharp/dull
sensation.
B36 C5 III
Four electrodes with current flow diagonal to the spinal column.
The crisscrossed electrode configuration allows: (1) a greater area to be treated and (2) current interference to occur
between the frequencies of the two circuits because of the diagonal pattern.
B37 C8 I
9-10 months.
The 9-10 month-old will be able to pull-to-stand, have all protective extension reactions, but since he is not yet walking,
may still exhibit plantar grasp in the standing position.
B28 C1 II
Postural reeducation, soft tissue mobilization, and dynamic stabilization.
This is a long term degenerative and postural dysfunction which is manageable with medication and proper physical
activity. Therefore, the most effective use of treatment time should emphasize regaining normal postural alignment and
functional ADL.
B39 C7 III
Ask about previous work and other experiences related to transferring individuals.
The therapists initial analysis of the learner should identify the current level of knowledge and skills about transfer
techniques. The therapist can then build on this information to plan appropriate educational goals and teaching methods.

333687839.doc

Answers

35

B40 C7 II
Skilled nursing facility
A skilled nursing facility is the best facility because the patient continues to require nursing care for his open wounds.
Initiation of physical therapy when this patient is able is also available. Discharge to home would be premature, as he is
unable to transfer. Custodial care involves medical or non-medical care that does not seek a cure. A rehabilitation hospital
is not appropriate at this time as the patient cannot actively participate in rehabilitation 3 hours per day.
B41 C2 I
Taste over the anterior tongue and having the patient raise his eyebrows and puff cheeks.
Bells palsy is a lower motor neuron lesion affecting the branches of the facial nerve, CN VII. Examination of the motor
function of the muscles of facial expression (i.e., raise eyebrows, show teeth, smile, close eyes tightly, puff cheeks) and
taste over the anterior tongue will reveal deficits of CN VII function. Taste over the posterior tongue is a function of CN IX
(glossopharyngeal). Strength of tongue protrusion is a function of CN XII (hypoglossal). Pupillary reflexes are a function
of CN II (optic).
B42 C6 I
Decreased trunk rotation with shorter steps.
Gait changes characteristic of Parkinsons disease include loss of arm swing and reciprocal trunk movements, shuffling
gait, and festinating cadence (an abnormal and involuntary increase in the speed of walking).
B43 C3 I
Orthostatic hypotension and dizziness
The adverse side effects that diuretics or calcium channel blockers have in common are orthostatic hypotension and
dizziness. These represent a safety risk during functional training and gait.
B44 C1 II
Within a few days following surgery to preserve tendon gliding.
Early passive and active assistive exercises promote collagen remodeling to allow free tendon gliding. When rehabilitation
is delayed by several weeks, adhesions form which restrict free tendon gliding.
B45 C8 I
Decreased pad and tenderness at the fracture sites.
Fracture healing in elderly clients is slower, and typically results in longer rehabilitation times than for young clients.
Hospitalization and surgery frequently cause disorientation and mental confusion as well as complications from prolonged
inactivity (e.g., pneumonia). Pain and tenderness are not decreased at the fracture site.
B46 C2 I
Visual confrontation test with a moving finger.
Visual field is examined using the confrontation test. The patient sits opposite the therapist and is instructed to maintain his
or her gaze on the therapists nose. The therapist slowly brings a target (moving finger or pen) in the patients field of view
alternately from the right or left sides. The patient indicates when and where he first sees the target. Distance acuity vision
is tested using a Snellen eye chart at a distance of 20 feet. Ocular pursuit is tested using a penlight moved in an H patter5n
to the extremes of gaze. Convergence is tested using a penlight and ruler; the patient keeps the penlight in focus as it
moves inward from a distance of 4 or 6 inches.
B47 C6 I
Common peroneal nerve
Sensation to the dorsum of the foot and big toe is supplied by the superficial peroneal nerve, a branch of the common
peroneal nerve. The sural nerve is a distal branch of the tibial nerve which supplies the back of the leg and the lateral side
of the foot and little toe. The medial calcaneal nerve is also a branch of the tibial nerve which supplies the heel and medial
sole of the foot. Phantom limb sensation (sensation of a limb that is no longer there) usually occurs in the immediate
postoperative phase and can be stimulated by external pressure (residual limb wrapping or rigid dressing). It typically
dissipates over time though some patients may experience the sensation of the rest of their life. This is a common finding
and should not interfere with prosthetic rehabilitation.
B48 C7 II
Accuracy of the measurements increases at higher saturation levels.
The result of the study indicates that the correlation between the two types of oximeters was high when oxygen saturation
levels were high (.89) but only moderate (.68) at low oxygen saturation levels.
333687839.doc

36

Answers

B49 C1 I
Anterior cruciate ligament.
The test shown in the figure is the Lachmans test to determine the integrity of the anterior cruciate ligament. The posterior
cruciate is examined using the posterior drawer and reverse Lachmans test. The medial meniscus is examined using the
McMurrays and Apleys tests. The ITP is tested using the Noble compression test.
B50 C1 II
CT scan
The primary concern of the therapist is to rule out strong suspicions of an upper cervical spine fracture. Plain films, already
taken, did not show any fracture which is not uncommon. The T-2 MRI and myelogram are not as specific for assessing
bony anatomy as the CT scan. CT scan is still preferred for assessing cortical bone especially spinal fractures.
B51 C5 II
Intense cold, burning, and aching, followed by numbness.
The stimulation of the cold receptors produces the perception of cold as the cold modality on the skin. The burning and
aching sensation is due to the intensity perceived as the tissue temperature decreases. As the transmission of the signals
from the skin receptors slow and finally ceases, numbness is produced.
B52 C8 III
Be informed of the right to make decisions regarding their medical care upon admission.
Advance Care Medical Directives are mandated by federal law, the Patient Self-Determination Act of 1990. Patients
receiving care from hospitals, nursing facilities, hospices, home health care agencies and health maintenance organizations
that receive federal Medicare or federal-state Medicaid funds must be informed on admission of their right to make
decisions regarding their medical care. The facility cannot alter these guidelines. Establishing mental competence is not
routinely required. Physicians cannot be mandated to give a contraindicated treatment; the patient can however refuse a
recommended treatment even if it leads to death.
B53 C2 II
Practice isolated small range quadriceps eccentric control work in standing and continue with the straight cane.
Eccentric quadriceps control work (closed-chair exercises) is indicated in order to reduce recurvatum. Open chain
exercises (Cybex, AROM) do not adequately address the functional demands of gait. The use of SBQC or hemi-walker will
not correct the problem. A KAFO is inappropriate to stabilize the knee which can be effectively stabilized using either an
AFO or Swedish knee cage. The use of an orthosis should be considered only as a last resort.
B54 C6 II
Progressive resistance training for the quadriceps.
A spinal cord injury at the level of L3 affects knee extensors. ASIA Scale D means the injury is incomplete with at least
half of the key muscles below the neurological level having a muscle grade of 3 or more. A weak knee will wobble or
buckle, going down stairs or ramps. It is the result of weak quadriceps or knee flexor contracture. Strengthening exercises
using progressive resistance training for the quadriceps is indicated. Biofeedback training may reduce knee extensor
spasticity but this may only increase knee instability and is not indicated in this case.
B55 C7 III
The patients employer when the condition is work-related.
Those individuals involved in the care of the patient, a legal guardian (power of attorney), or the patients payer have legal
right to information regarding a patients care without obtaining the patients consent for releasing information.
B56 C6 III
Increasing the angle between the seat pan and backrest to between 90-110 degrees.
Maximal reduction of lumbar disc pressures can be achieved by increasing the ankle between the seat pan and backrest to
between 90-110 degrees, using armrests for support or adding a lumbar support. Combining the effects of all three provides
the best solution.
B57 C1 I
Suprascapular nerve
Microtrauma to the suprascapular nerve can occur with repetitive activities involving shoulder cocking and follow
through resulting in inflammation and muscle weakness of the muscles supplied by the suprascapular nerve (the
supraspinatus and infraspinatus muscles).

333687839.doc

Answers

37

B58 C7 III
Needs assessment performed by a physical therapist.
Other professionals (physician, nurse) can refer a patient for evaluation, but determination of need must be performed by a
licensed physical therapist.
B59 C2 II
Repetition of movements and positions that provoke dizziness and vertigo.
In patients with unilateral vestibular pathology, habituation training (use of positions and movements that evoke symptoms)
will encourage the vestibular system to recalibrate. Good recovery can generally be expected with gradual progression of
exercises. Hallpike maneuver is used for assessment, and is not a set of exercises.
B60 C6 I
Too little tension in the extension aid.
Excessive heel rise during early swing can be caused by inadequate knee friction or too little tension in an extension aid.
B61 C4 II
Give the patient water and notify the physician immediately.
This patient is exhibiting signs of dehydration. His confusion is a poor sign and requires immediate action: get some fluids
into him and notify his physician immediately.
B62 C3 II
1.5 mm of downsloping ST segment depression.
The upper limit for exercise intensity prescribed for patients post MI is based on signs and symptoms. Of the choices, only
ST segment depression (greater than 1.0 mm of horizontal or downsloping depression) is a significant finding,
representative of myocardial ischemia. Both HR and BP are expected to rise (the levels of 140 and 140/80 are significant
for most patients). The appearance of a singe PVC is also not significant since PVCs can occur in individuals without
cardiac abnormalities.
B63 C8 II
Neutral warmth
The infant is still too developmentally immature for any of the stimuli other than neutral warmth. Neutral warmth is a
calming stimulus; all of the other choices would likely increase arousal of the infant.
B64 C2 I
Ipsilateral weakness and loss of position sense and vibration below the lesion level with contralateral loss of pain and
temperature sensation.
Brown-Sequard syndrome is a hemisection of the spinal cord characterized by ipsilateral weakness and loss of position and
vibration sensation below the level of the lesion; there is also contralateral loss of pain and temperature sensation a few
segments below the level of the lesion. Other choices describe an anterior cord syndrome (choice A); central cord
syndrome (choice B); or sacral sparing (choice C).
B65 C7 II
Demonstrate the task, then practice with the patient.
To ensure optimal motor learning, first demonstrate the task at ideal performance speeds. This provides the patient with an
appropriate reference of correction (cognitive map) of the task. Then use guided practice with the patient to ensure safety
and successful performance. Caregivers should become involved only after initial practice of the task with the patient and
the safety of the patient can be assured.
B66 C7 III
Home health agency rehabilitation services.
Medicare Part B covers doctors services, outpatient services, physical therapy private practice services, and durable
medical equipment. Rehabilitation services from a home health agency are covered under Medicare Part A.
B67 C6 I
Socket is aligned too far forward or tilted anteriorly.
In a PTB prosthesis, the socket is normal aligned in slight flexion to enhance loading on the patellar tendon. If it is aligned
incorrectly (too far anterior or excessively flexed), it will result in excessive knee flexion in early stance.

333687839.doc

38

Answers

B68 C3 II
Sustained maximal inspiration training with an incentive spirometer.
Sustained maximal inspiration is an appropriate part of a plan of care. Percussion and shaking are indicated, but not to both
sides, since pneumonectomy removes the entire lung. Chest tubes are not a contraindication to the use of these techniques.
There is no point in teaching breathing exercises to the right thorax as there is no lung underneath.
B69 C6 II
A high back
At T9-T10 this patient has partial innervation of the abdominals (innervated T6-T12) and full innervation of the upper
extremities. A manual wheelchair with a standard back height (midscapular height) is appropriate for this patient. If the
patient chooses to engage in sports, a lower back can improve functional ability. A high back wheelchair is appropriate for
patients with poor trunk stability or extensor spasms. Swing-away footrests and removable arm rests are necessary for ease
of transfers and can be removed for sports activities as necessary.
B70 C1 II
Finkelsteins test
Finkelsteins test is specific for reproducing the pain associated with deQuervains tenosynovitis of the abductor pollicis
longus and extensor pollicis brevis. Froments sign is used to identify ulnar nerve dysfunction. Phalens test identifies
median nerve compression in the carpal tunnel. Craigs test identifies an abnormal femoral antetorsion angle which you
hopefully eliminated first!
B71 C8 II
A pommel to keep hips abducted.
Hip abduction should be facilitated in sitting while hip adduction, extension of the hips, and plantarflexion of ankles should
be inhibited.
B72 C2 I
Sensory losses and sensory organization of balance.
A critical component of balance control is sensory input from somatosensory, visual and vestibular receptors and overall
sensory organization of inputs. Initial examination should address these elements before moving on to assess the motor
components of balance (e.g., postural synergies). The Clinical Test for Sensory Integration in Balance (Shumway-Cook,
Horak) is an appropriate instrument. Cardiovascular endurance and level of dyspnea during functional transfers are
appropriate elements to examine but should occur after key elements of balance are examined (sensory components and
integration; synergistic elements).
B73 C7 III
The PT who is solely responsible assessing the competence of the PTA under their supervision.
The PT is solely responsible for assessing the competence of all personnel under their supervision. Furthermore, it is the
responsibility of the supervising PT to exercise sound judgment when delegating responsibility to less skilled personnel.
B74 C7 III
Complete the written discharge summary at another time and examine the patient.
The physical therapist assistant can perform assigned physical therapy interventions under the onsite supervision of PT. It is
the obligation of the physical therapist to examine this scheduled patient in a timely manner and determine the plan of care.
This task cannot be delegated.
B75 C1 I
Ankle plantarflexion and anterior pelvic tilt.
A common contributory problem or correlated motion for genu recurvatum is ankle plantarflexion due to shortened
gastrocnemius muscles. Alterations occurring up the kinetic chain include anterior pelvic tilt to maintain the center of
gravity over the feet.
B76 C1 I
L4 nerve root compression
The positive findings are consistent with a L4 nerve root compression. Weakness of only one muscle group is not a
common finding for DJD or SI dysfunction.

333687839.doc

Answers

39

B77 C4 I
Pre and post activities, using the non-shunt arm.
A dialysis shunt would interfere with taking BP. Use the non-shunt arm. Pre and post exercise measurements are
appropriate; taking BP during walking would result in inaccurate measurements.
B78 C6 II
Three-point
A three-point gait pattern in which both crutches and the involved leg are advanced together is the safest gait pattern to
ensure partial or nonweightbearing following a lower extremity fracture. Two-point and four point are weight bearing gaits.
B79 C7 II
Level 3 case-control study provides limited confidence in the effectiveness of TENS.
The experimental design includes random assignment into experimental group (receives treatment) or control group (no
treatment). Sacketts Levels of Evidence and Grades of Recommendation are used to define the degree of confidence and
scientific rigor of the research. Level I studies require large randomized controlled trials (RCTs) and provides maximum
confidence in the results (Grade of A). Level II studies are cohort studies and smaller RCTs and provide good evidence
(Grade of C). Level 3 studies, case-control studies (as in this example) allow some limited confidence in the findings
(Grade of B). Level 4 studies are poor quality case-series (Grade of C) while Level 5 studies are based on expert opinion
only (Grade of D).
B80 C3 II
Stop the exercise and notify the physician immediately.
A run of three or more PVCs occurring sequentially is ventricular tachycardia. The rate is very rapid resulting in seriously
compromised cardiac output. This potentially an emergency situation that can deteriorate rapidly into ventricular
fibrillation (no cardiac output) and cardiac arrest.
B81 C2 I
Visual agnosia.
All of the choices are indicative of perceptual dysfunction. This patient is most likely suffering from visual agnosia which
is an inability to recognize familiar objects despite normal function of the eyes and optic tracts. Once the wife talks with
him, he is able to recognize her by her voice. Ideational apraxia is the inability to perform a purposeful motor act, either
automatically or upon command. Anosognosia is the frank denial, neglect or lack of awareness of the presence or severity
of ones paralysis. Somatognosia is an impairment in body scheme.
B82 C8 I
Buffered aspirin
There is a long list of risk factors that may increase an elderly patients risk of falling. Psychotropic drugs (Elavil, Haldol)
can cause postural hypotension and are strongly linked with falls. Digoxin is used in treating arrhythmias and can induce
dizziness, fatigue and confusion. Nitroglycerin is an antianginal drug that can also cause postural hypotension. Of the
medications listed only Buffered aspirin, an NSAIDS, is not likely to increase fall risk (adverse side effects include GI
disturbances such as stomach pain, heartburn, nausea and vomiting).
B83 C1 II
Use of a resting splint at night
The symptoms are suggestive of plantar fasciitis. The focus of patient management should be on decreasing the irritation to
the plantar fascia. This is most effectively done with a resting night splint. Modalities to reduce pain offer some
symptomatic relief; however, the pain is not constant. Strengthening the dorsiflexors will not change irritation to the
plantar fascia. A customized orthosis may be necessary, at a later time, if primary symptoms do not resolve after early
management.
B84 C6 III
Grasp the handrims posteriorly and pull them forward abruptly and forcefully.
A wheelie can be assumed by having the patient place his hands posterior on the handrim and pulling them abruptly and
sharply forward. If the patient is unable to lift the casters in this manner, he can throw his head back forcefully when he
pulls his handrims. An alternate technique is to grasp the handrims anteriorly, pull backward, then abruptly and forcefully
reverse the direction of pull. The therapist can assist by steadying the chair at the patients balance point until he learns to
adjust his position through the use of handrim movements forward and backward.

333687839.doc

40

Answers

B85 C3 I
Amyotrophic lateral sclerosis with dysphagia and diminished gag reflex.
Aspiration pneumonia results from an abnormal entry of fluids or matter (including food) into the airways. A patient with
ALS with an inability to swallow (dysphagia) and diminished gag reflex is most susceptible to aspiration pneumonia.
Others listed may be susceptible to other forms of pneumonia or even, though less likely, aspiration pneumonia.
B86 C6 II
Partial squats progressing to lunges.
Closed-chain exercises are the most appropriate in this example because of the patients difficulty descending stairs.
Moving the body over a fixed distal segment provides loading to muscles, joints, non-contractile soft tissue while
stimulating the sensory receptors needed for stability and balance. Open-chain exercise (all other choices) while improving
strength does not adequately prepare an individual for functional weight bearing.
B87 C2 I
Low lumbar (L4-L5)
Patients with low lumbar lesions (L4-L5) can be independent and functional with bilateral ankle-foot orthoses and cane.
Patients with higher lesions can learn to ambulate with orthoses and crutches but exhibit a high rate of orthotic rejection in
favor of primary wheelchair mobility. Rejection is due to the high levels of energy expenditure during ambulation.
B88 C5 II
Paraffin
Paraffin bath will provide circumferential heating of the hands and fingers and will aid in softening the skin. Active
exercise, including FES, would be more effective after the application of paraffin as tissue extensibility and pliability would
be increased. Hot packs or ultrasound using direct contact would not completely cover the area to be treated.
B89 C6 II
Front wheel rolling walker that folds.
A rolling walker will provide added stability while maintaining gait as a continuous movement sequence. The additional
benefit of a folding walker facilitates mobility in the community.
B90 C1 III
Forward, producing posterior tilting of the pelvis.
Contraction of the lower abdominals results in posterior tilting of the pelvis and can be achieved with forward or anterior
movement of the therapy ball.
B91 C2 I
A peripheral nerve injury.
This patient is exhibiting signs and symptoms of lower motor neuron injury (hypotonia, hyporeflexia, paresis, neurogenic
atrophy). The presence of muscle fasciculations is a hallmark sign of LMN injury. Upper motor neuron lesions (cortex or
pyramidal tracks) would result in hypertonicity (Hypotonicity initially during shock), hyperreflexia, generalized paresis,
and variable disuse atrophy. Guillain-Barr is a LMN condition that produces symmetrical and ascending signs.
Extrapyramidal signs (involuntary movements) are not evident in this case.
B92 C2 I
The spinal nucleus of CN V
Syringomyelia is a disease of the spinal cord characterized by the development of cysts or cavities within the cord. The
sensory losses exhibited by this patient are characteristic of spinal cord involvement. The main sensory nucleus of CN V,
trigeminal nerve, is located in the pons and connects inferiorly to the spinal nucleus below which extends through the whole
length of the medulla into the upper spinal cord as far as C2. Sensations of touch and pressure are served by the main
sensory nucleus of CN V (intact in this case) while pain and temperature are served by the inferior spinal nucleus (impaired
in this case).
B93 C3 II
Standard postural drainage for the lateral basal segment, left lower lobe.
The appropriate postural drainage for the lateral basilar segment of the left lower lobe is in sidelying position with the head
of bed tipped in full Trendelenburg. With a respiratory rate of 28 at rest and borderline SaO 2 values on 4 liters of oxygen,
modification of the position is in order.

333687839.doc

Answers

41

B94 C8 I
His symptoms are intermittent.
Acutely ill, hospitalized elderly patients frequently exhibit delirium, a fluctuating attention state. Patients demonstrate a
fluctuating course with symptoms of confusion that alternate with lucid intervals. Sleep/wake cycles are disrupted and
confusion is typically worse at night. All other choices are signs of dementia.
B95 C7 II
Analysis of variance.
Analysis of variance is a parametric statistical test used to compare three or more treatment groups (in this example, in
water, on land or combined exercise groups) on a measure of the dependent variable (joint effusion girth measurements) at
a selected probability level. Analysis of covariance compares two or more groups but also controls for the effects of an
intervening variable.
B96 C2 II
Demonstrate and gesture to get the idea of the task across.
Wernickes aphasia (fluent aphasia) is a central language disorder in which spontaneous speech is preserved and flows
smoothly while auditory comprehension is impaired. Demonstration and gesture (visual modalities) offer the best means of
communicating with this patient. The family may not better understand the patient than the therapist.
B97 C6 I
Insufficient knee friction.
Terminal swing impact refers to the sudden stopping of the prosthesis as the knee extends during late swing. Possible
causes can include insufficient knee friction or too much tension in the extension aid. Additionally, if the patient with an
amputation fears the knee will buckle at heel strike, he can use forceful hip flexion to extend the knee.
B98 C4 II
Plastic cervical orthosis and axillary splints utilizing an airplane position.
The common deformity for an anterior neck is flexion; the appropriate positioning device is a firm rigid plastic cervical
collar that stresses extension. The common deformity of the shoulder is adduction and internal rotation; the appropriate
position device is an axillary or airplane splint that stresses abduction, flexion and external rotation. Choices that involve
hand splints should be ruled out immediately as there is no mention of burns to the hands. A CTLSO would prevent neck
flexion but it does not deal with the potential shoulder deformities. The CTLSO could also restrict breathing and enhance
the risk of pneumonia.
B99 C1 I
Compression of the common peroneal nerve.
Cutaneous neural innervation to the great toe is via the deep peroneal nerve which is a branch of the common peroneal
nerve. Motor problems of toe extension can also be traced to common peroneal nerve compression. Compromise of the
tibial artery could result in paresthesias of the plantar surface of the foot and loss of ability to flex the toes. The cast must
be removed or adjusted immediately.
B100 C2 III
Bridging with both arms positioned in extension at the sides.
Bridging, with arms positioned at the sides, is a symmetrical activity and would do little to bring attention to the involved
hemiplegic side. All of the other activities involve crossing the midline, and focus on moving toward or incorporating the
involved left side.
B101 C7 III
Provide the family Medicare notification of noncoverage information, and carefully explain it and their options, which
could include playing for the care out-of-pocket.
Medicare requires that physical therapy is skilled, appropriate and demonstrates progress towards reasonable functional
goals. Despite the fact that Medicare requires that the physician responsible for the patient certify the necessity and
appropriateness of physical therapy, that does not absolve the physical therapist from the responsibility of determining
whether physical therapy is appropriate. That is the sole responsibility of the PT.

333687839.doc

42

Answers

B102 C3 II
Call the patients physician and report signs of theophylline toxicity.
Theophylline is a bronchodilator used to reverse airway obstruction. The combination of symptoms of irregular heart rate,
feeling jittery and gastric upset is consistent with theophylline toxicity. As theophylline toxicity can cause arrhythmias and
seizures, the patients physician should be notified by the physical therapist rather than wait for the patient to return home
to call the physician. It is also likely that a blood test will be needed to check the theophylline toxicity level and this could
be done at the facility. Ventolin (Albuterol) is a bronchodilator used in the treatment of asthma or COPD. Amcort
(Triamcinolone) is an anti-inflammatory agent used to manage bronchial asthma.
B103 C8 I
Atlantoaxial subluxation with spinal cord impingement
Ligamentous laxity is a hallmark of Down syndrome and can lead to atlanto-axial subluxation with spinal cord
impingement. This is a medical emergency situation. Decreased muscle strength and diminished DTRs are the signs of
such a dislocation from loss of cord function. Children with Down syndrome often have low tone and a number of
orthopedic abnormalities. In this case, the increasing symptomatology (changes in strength, neck pain, limited neck
motion) is significant for a developing subluxation.
B104 C2 I
Passive range of motion while emphasizing scapulohumeral rhythm.
Shoulder pain in hemiplegia may be caused by a number of different factors affecting subluxation (Choices A, B, and C).
An appropriate treatment intervention involves PROM with careful attention to maintaining scapulohumeral rhythm.
B105 C6 I
Weakness or severe spasticity of the quadriceps
Severe spasticity of the quadriceps will pull the knee strongly into extension and recurvatum. The patient with a weak
quadriceps muscle will compensate by fixing the knee in full extension (recurvatum) during stance rather than maintaining
the knee in a small amount of knee flexion at midstance.
B106 C4 II
Use of an AFO with heel pressure relief.
The AFO helps to prevent plantarflexion contractures while the heel pressure relief prevents further damage and promotes
healing. The pressure ulcer, based on the examination findings, is stable and needs to be monitored not debrided. Arterial
bypass grafts are needed if circulation is compromised. There is no indication that this is the case.
B107 C7 II
Leg ergometry is highly correlated with RPE while arm ergometry is only moderately correlated.
In correlational studies, high correlations range from 0.7 to +1.00 while moderate correlations range from 0.35 to 0.69.
Common variance is a representation of the degree that variation in one variable is attributable to another variable and is
determined by squaring the correlation coefficient.
B108 C1 II
Immediately return the patient to the referring physician with documentation indicating that treatment was ineffective.
In this case, it would be best to send the patient back to the referring physician with an explanation of what was done; the
ineffectiveness of the treatment, and any suggestions for further follow up. The therapist should not continue to treat a
patient if the therapist feels no further benefit would be derived by continuing care because it contradicts the APTAs
Guidelines for Professional Conduct.
B109 C5 III
Continuous US @ 3 MHz
Since the ankle sprain is chronic in nature, the goal is to decrease pain and increase the ROM (tissue extensibility).
Increasing the tissue temperature will accomplish these objectives. This will require the heating effects of ultrasound
produced by continuous ultrasound. Because the largest structures are close to the skin surface, higher frequencies are used
so that the acoustic energy will be absorbed (attenuated) in the superficial layers.
B110 C2 III
Facilitation of early movement in synergistic patterns followed quickly by movement training in out-of-synergy patterns.
The focus of NDT therapy (Bobath Approach) following stroke is to encourage functional return of normal patterns of
movement. To that end, obligatory synergistic patterns are never encouraged and from the earliest onset of active
movement, activities are designed to break up the abnormal synergies.
333687839.doc

Answers

43

B111 C8 II
Bilateral KAFOs
The knees would not need to be braced in moderate spastic diplegia, but tone reducing AFOs might be indicated. A
posterior walker and prone stander may both enhance function.
B112 C6 II
Strengthen the hip abductors on the right side.
The lateral trunk bending (Trendelenburg gait) is the result of weak hip abductors on the right (a common problem for
patients recovering from stroke). The other choices do not address the problem of lateral trunk bending.
B113 C1 II
Decreased loading of the knee by the quadriceps femoris muscle.
Baseball catchers must make forceful contractions of the quadriceps muscles each tome they stand up to throw the ball to
the pitcher. This may precipitate Osgood Schlatter dysfunction in the adolescent boy. Early intervention of this condition
focuses on reduction of the loading by the quadriceps but still retaining normal lower extremity function.
B114 C3 II
Calcium alginate dressings.
Wounds with moderate to high exudate benefit from calcium alginate dressings. The dressings absorb large amounts of
exudate and form a gel which maintains the moist wound environment while maintaining good permeability to oxygen.
B115 C1 I
Centralized gnawing pain with loss of postural control during lifting activities.
Discal degeneration without nerve root compression would likely be exhibited as a centralized gnawing pain with loss of
proprioception.
B116 C2 II
Involving the patient in decision making and monitoring for safety.
As patients with TBI recover, structure and guidance must be gradually reduced and patient involvement in decision
making increased. Safety must be maintained while increasing levels of independence are fostered. Patients at Stage VII of
recovery often exhibit rote movements (robot syndrome) indicative of the highly structured training utilized for patients
during earlier stages of recovery.
B117 C6 II
Use a toe wedge
A festinating gait is an abnormal and involuntary increase in the speed of walking in an attempt to catch up with a displaced
center of gravity due to the patients forward lean. The most appropriate intervention would be to use a toe wedge which
would help to displace the patients center of gravity backward. Increasing cadence or stride length would only serve to
increase, not decrease, the problem.
B118 C6 III
High intensity workloads for short durations.
High intensity exercises for shorter durations (less than 20 repetitions) are needed to train the highly adaptable fast-twitch
IIa fibers.
B119 C2 I
Sympathetic nerve damage consistent with Horners syndrome.
In TBI, the oculomotor nerve is vulnerable from a herniated uncus. Compression on the nerve can produce Horners
syndrome resulting in symptoms of constriction of the pupil (miosis), slight drooping of the eyelid (ptosis), vasodilation of
the skin arterioles, and loss of sweating (anhydrosis). All of these symptoms are the result of an interruption of the
sympathetic nerve supply to the head and neck. An Argyll-Robertson pupil is a small pupil of fixed size and not reactive to
light that is usually caused by interruption of fibers to the parasympathetic nuclei (Edinger-Westphal nuclei) of the
oculomotor nerve.
B120 C8 II
Household ambulation with a reciprocating gait orthosis (RGO) and Lofstrand crutches.
A child with a very high level myelomeningocele will be able to ambulate for limited (household) distances with an RGO
and Lofstrand crutches. Physiological benefits include improved cardiovascular and musculoskeletal function. The child
will not be able to be a community ambulator because of the high energy expenditure necessary with this level of lesion.
333687839.doc

44

Answers

B121 C7 II
Placebo effect
In placebo effect, a subject responds to a sham treatment (in this example the patient received a pill that did not contain the
drug being tested) with positive effects (patient reports she feels much better, moves easier. The Hawthorne effect is the
subjects knowledge that she is part of a study and that alone might affect her responses. Sampling bias in a national study
is unlikely and wouldnt explain her response.
B122 C3 I
This arterial blood gas shows an increased pH which is an alkalosis. It is caused by the low carbon dioxide level, therefore,
it is a respiratory alkalosis. There is no compensation, since the bicarbonate ion is at the baseline level of 24 mEq/dL.
B123 C4 III
Avoid exercise during periods of peak insulin activity
The patient should monitor blood glucose levels frequently when initiating an exercise program and avoid exercise during
periods of peak insulin activity (2-4 hours after injection). A carbohydrate snack should be eaten before and during
prolonged exercise bouts. The physician can also decrease the insulin dose by 1-2 units. Exercise should begin with daily
sessions, 20 minutes twice a day. The therapist should use RPE in addition to HR to monitor exercise intensity. (Source:
ACSMs Guidelines for Exercise Testing and Prescription, 5th ed.).
B124 C8 II
Will likely be resistant to activity training if unfamiliar activities are used.
Activity training is most likely to be successful if done with familiar activities. A patient with Alzheimers disease cannot
be trusted to safely perform independent ADLs or functional mobility skills. Memory for past events may be retained
initially but eventually all memory becomes impaired.
B125 C2 III
Focus on knowledge of results and visual inputs.
During the early stage of motor learning (cognitive stage) learners benefit from seeing the whole task correctly performed.
Dependence of visual inputs is high. Developing a reference of correctness (knowledge of results) is critical to ensure early
skill acquisition (cognitive mapping). Focus on proprioceptive inputs is important during the middle (associative) stage of
motor learning. Delayed feedback is appropriate during later learning.
B126 C6 III
Pad placed proximal to the metatarsal heads.
Compression of the digital nerves in the forefoot results in sensory symptoms of pain and paresthesias (metatarsalgia). It is
typically the result of excessively tight shoes. The best intervention is to wear larger shoes with a metatarsal pad placed
proximal to the metatarsal heads to elevate the transverse (anterior) arch and separate the metatarsals. Custom orthotics can
also be molded to decrease load. Wearing of high heels should be discouraged. Stretching of plantarflexors may also be
helpful. A scaphoid pad and Thomas heel are used to support the longitudinal arch and prevent pes valgus and
metatarsalgia.
B127 C1 II
Modalities to reduce pain and inflammation as the result of subdeltoid bursitis.
Because pain occurs with both AROM and PROM, bursitis is the most likely cause of dysfunction. Initial intervention
should focus on reducing pain and inflammation. Modalities are the best choice.
B128 C4 I
Motor ataxia and paresis with pronounced gait disturbances.
Alterations in memory, confusion, and disorientation are characteristic of AIDS dementia complex, a common CNS
manifestation of HIV infection. Motor deficits may include ataxia, paresis with gait disturbances, and loss of fine motor
coordination. Patients may also develop peripheral neuropathy with distal pain and sensory loss. Paraparesis, not
paraplegia might be a finding.
B129 C7 III
The applicants marital status.
It is illegal to ask potential applicants their marital status.

333687839.doc

Answers

45

B130 C8 III
Femoral antetorsion and medial knee stress.
W sitting is a stable and functional position, but may cause later orthopedic problems of femoral antetorsion and knee
stress. Children with Down syndrome typically exhibit low tone and hyperextensibility. W sitting is not likely to affect low
tone or reflex activity. Hip subluxation is unlikely.
B131 C2 I
Dorsal column/lemniscal pathways or somatosensory cortex.
Discriminative touch is carried in the posterior white columns (fasciculus cuneatus for the upper extremity and fasciculus
gracilis for the lower extremity). The long ascending tracts cross the medulla (sensory decussation) and form the medial
lemniscus which then travels to the thalamus (ventral posterolateral nucleus) and finally to the cortex (post central gyrus).
Loss of two-point discrimination could result from an insult affecting any of these component parts. Parietal lobe or
internal capsule lesions are the most common sites.
B132 C3 II
Daily, using interval training for 10 to 15 minute periods.
An appropriate initial exercise prescription for a patient with a history of CHF and claudication pain in the right calf should
include low intensity exercise (walking), low to moderate duration (10-15 minutes), and higher frequencies (daily). The
exercise session should carefully balance activity with rest (interval training).
B133 C7 III
A statement ensuring the subjects commitment to participate for the duration of the study.
Subjects may refuse to participate and withdraw from a study at any time. Recognition of this element must be clearly
stated in a valid informed consent.
B134 C4 II
Increasing non-weightbearing and stabilization using a lumbosacral orthosis.
Ligamentous laxity and pain during pregnancy secondary to hormonal influences (relaxin) most commonly affects the
sacroiliac joint. This ligamentous laxity continues to occur for up to 3 months after pregnancy, and leaves the pelvic area
vulnerable to injury. Sacroiliac pain is aggravated by prolonged weight bearing and stairs. The ipsilateral flexion kinetic
test (Gillet Test: stand on one leg while flexing the opposite hip) assesses mobility of the innominate and ability of the
ipsilateral innominate to rotate posteriorly. All the interventions listed can be helpful in this situation except for increasing
non-weightbearing time and use of a lumbosacral orthosis which are not mobilizing techniques or activities designed to
restore optimal alignment and mobility.
B135 C5 III
Skin and electrodes.
Electrodes must be checked regularly for signs of wear or cracking which would make the delivery of electrical current
either ineffective and/or unsafe. Repetitive long-term use of electrodes might produce cracking or uneven wear, which
could develop into hot spots of increased current density. The increased current density along with continued placement
and removal of the electrodes could cause skin irritation and breakdown.
B136 C6 III
Improve positioning for pressure relief or extensor spasms.
A tilt-in-space wheelchair has the entire seat and back tilted backwards as a unit and is prescribed to provide additional
postural support, pressure relief, or to prevent extensor spasms from throwing the patient out of the chair.
B137 C8 II
Practice walking in areas of high illumination and low clutter.
Effective intervention strategies for the elderly patient with low vision include ensuring adequate lighting. Vision and
safety decrease dramatically in low lighting or with bright glare from sunlit windows. Reducing clutter in the home is also
an important strategy to improve safety during ambulation. Color coded stairs might be of help if they were well lit and if
strong colors, not pastels, were used.
B138 C3 I
Angina of increasing intensity that is unresponsive to the nitroglycerin or rest.
Preinfarction or unstable angina pectoris is unrelieved by rest or nitroglycerin (measures that typically reduce most angina).
The pain is described as increasing in intensity. Unstable angina is an absolute contraindication to exercise.

333687839.doc

46

Answers

B139 C2 III
Foot tapping in a sitting position.
The typical lower extremity synergies are effectively broken up with (1) kneeling and bridging (combines hip extension
from the extensor synergy with knee flexion from the flexion synergy) and (2) rolling using PNF D1 flexion patterns
(combines flexion from the flexion synergy with adduction from the extension synergy). Foot tapping in the sitting position
is an in-synergy activity (dorsiflexion with hip and knee flexion).
B140 C7 III
Dependency on others for all transportation needs.
The overall requirements and qualifying criteria for homebound care are: 1. Care must be provided in the home; 2. The
patient is confined to the home and requires a considerable and taxing effort to leave; and 3. If the patient leaves home he
or she is still considered homebound if the absences from home are to receive medical treatment, radiation, dialysis, etc.
The fact that the patient is dependent on others for transportation does not meet the homebound criteria.
B141 C6 I
A stiff knee mechanism
A prosthesis that is too long or has a knee mechanism that is too stiff will cause the patient to circumduct or abduct the
prosthesis in order to clear the prosthesis and get through swing phase. If it is not compensated, a fall may result. The
other problems will not result in a circumducted gait pattern.
B142 C4 II
Notify the physician immediately.
The risk of metastatic disease is present; the therapist should notify the physician immediately. Monitoring or modifying
the plan of care to reduce pain should be considered only after consultation with the physician. If metastatic disease is
present, the ultrasound would be contraindicated. Ambulating in the parallel bars exerts the same weightbearing forces
through the upper extremities as a walker.
B143 C5 II
Cold/intermittent compression combination with the limb elevated.
The application of cold to decrease pain along with intermittent compression with elevation to facilitate fluid drainage
provides the best intervention. Contrast baths and whirlpool place the ankle in a dependent position which might tend to
increase edema.
B144 C1 I
Mid-cervical region.
The patient has symptoms (diminished reflex) of a possible left C5 nerve root compression in the mid-cervical spine. Any
reflex change suggests nerve root irritation or compression. Lateral epicondylitis frequently involves the extensor carpi
radialis brevis which is innervated by spinal nerves emanating from the mid-cervical region.
B145 C3 II
Walking, intensity set at 50% target heart rate.
Obese individuals are typically sedentary with lower initial levels of physical conditioning. The initial exercise prescription
should focus on a lower intensity exercise progressing to longer durations. Higher intensity exercise (75% or 85% of heart
rate maximum) should be avoided initially. Jogging is also too intense and may yield additional orthopedic problem.
B146 C1 I
Contracted hip flexors.
Patients are less active after surgery, spend less time in standing and more time in sitting. The iliopsoas muscles become
shortened with increased time in sitting. The contracted iliopsoas limits the patients ability to extend the hip which
effectively shortens the stride length on the affected side.
B147 C8 III
Slowly stroke the finger extensors in a distal to proximal direction.
Slowly stroking the finger extensors will help to facilitate the finger extensors to allow the child to release the food into the
mouth.

333687839.doc

Answers

47

B148 C2 II
Low intensity general conditioning exercises.
Patients with ALS are at risk from overwork injury to weakened, denervated muscles (a characteristic of LMN disease).
While preventing further deconditioning is an important goal, the exercises prescribed should be of mild to moderate
intensity and carefully balanced with rest.
B149 C6 II
The distance of the load from the base of the spine
Manual lifting biomechanical models have demonstrated high lumbar spine moments especially when the load is not held
close to the body.
B150 C1 II
Anterior inferior translatory glides.
The diagnosis is left shoulder adhesive capsulitis. Because the restriction of motion occurs in abduction, flexion, and
external rotation, anterior inferior translatory glides are most appropriate for this patient based on the concave-convex
mobilization concept.
B151 C8 I
Impairments in short-term memory
Elderly patients with memory impairments typically demonstrate intact immediate recall (e.g., can repeat words);
impairments are often noted in memory for recent events, e.g. Why did I come into this room? or Who came to see me
yesterday?. Long term memory is usually intact.
B152 C2 I
A positive Hallpike maneuver.
The test described is the Hallpike maneuver. It is positive because the change in position produced the patients symptoms.
While the CTSIB and the sharpened Romberg are also used to assess vestibular function, they can be expected to produce
symptoms of dysequilibrium (loss of balance).
B153 C7 III
Have the patient practice walking in varying environments.
This patient demonstrates the associative phase of motor learning (errors are decreasing and movements are becoming
organized). It is appropriate to gradually progress this patient toward ambulating in a more open (varied) environment.
Continuous feedback may improve performance but in the long run slows down learning. Practice until errors are
extinguished or early intervention, also represent inappropriate strategies for the associative stage of learning.
B154 C3 III
Autogenic drainage for secretion removal once or twice daily.
Proper at home interventions include: secretion removal techniques including an a capella device or manual techniques
performed by an adult, and exercise are appropriate interventions for a child with cystic fibrosis. Autogenic drainage which
can be helpful in clearing secretions but is not appropriate for a young child as it relies on independent ability to monitor
secretion build up in the airways and to make appropriate choices of clearance.
B155 C4 II
Kegel exercises several times a day.
Symptoms of stress incontinence can be successfully managed through a variety of techniques. Pelvic floor exercises (Keel
exercises) are the mainstay of treatment and must be performed daily, several times a day in order to be effective.
Biofeedback and E-Stim offered weekly or 3 times a week are not likely to be effective due to insufficient frequency. A
voiding schedule does not address the primary impairment.
B156 C2 I
Posterior inferior cerebellar artery.
This patient presents with lateral medullary (Wallenbergs) syndrome which can result from occlusion of the posterior
inferior cerebellar artery (PICA) which is usually a branch of the vertebral artery. It involves the descending tract and
nucleus of CN V, the vestibular nucleus and its connections, CN IX and CN X nuclei or nerve fibers; cuneate and gracile
nuclei; and spinothalamic tract. An internal carotid artery stroke produces symptoms of combined middle cerebral and
anterior cerebral artery strokes. The symptoms in this case clearly indicate brainstem involvement, not cortical
involvement.

333687839.doc

48

Answers

B157 C5 II
Cathode placed in the wound.
It is purported that the bactericidal effect produced by negative current is a result of substrate depletion or alteration of the
internal processes of the microorganisms. Neutrophils are also attracted to the wound area by chemotaxis to purge the
bacteria. The cathode should be placed directly in contact with the wound to cover as much treatment area as possible.
B158 C3 III
Moderate intensity, (8-9 METS) and can be easily alternated with walking at 5 METs.
Running on a flat surface (12 min. per mile) is a moderate intensity activity with an energy cost around 8.7 METs. It can be
easily alternated with the lower intensity activity of walking (5 METs) to achieve suitable intensities for individual patients.
B159 C4 II
Initiate functional training consistent with her level of injury.
A conversion disorder (hysterical paralysis) represents a real loss of function for the patient. The therapist should treat this
patient the same as any patient with spinal cord injury with similar functional deficits. A psychologist or psychiatrist is best
able to help the patient understand the cause of her paralysis. The therapist should be empathetic, however, counseling
should not be the main focus of intervention in PT.
B160 C7 III
Refused to treat this patient.
Malpractice is considered professional negligence as a result of wrongs or injuries that may occur through
professional/patient relationships. Negligence is the failure to do what a reasonable practitioner would have done or not
done in a similar circumstance. A supervisors request to treat a patient can be refused if the therapist feels it is outside the
scope of his/her expertise.
B161 C8 I
Continuous tremulousness.
Continuous tremulousness is an abnormal finding, but occasional tremulousness is not. All the other choices are normal
findings in a newborn infant.
B162 C6 I
Sacral sitting and sliding forward in the chair.
Excessive leg length on a wheelchair can result in sliding forward in the wheelchair to reach the foot plate. This movement
results in a posterior tilt of the pelvis and sacral sitting.
B163 C4 I
Exudate in the wound.
The specimens must be collected from the wound site with a minimum of contamination by material from adjacent tissues.
The margins of cutaneous lesions or pressure ulcers are usually contaminated with environmental bacteria. Using the
dressing for a specimen sample would also contain contaminated tissues.
B164 C3 III
Both HR and systolic/diastolic BP will be higher.
Arm ergometry uses a smaller muscle mass than leg ergometry resulting in a lower maximal oxygen uptake. In upper
extremity exercise both HR and BP will be higher than for the same level of work in the lower extremities.
B165 C7 II
t-test.
The data in this study is interval data (values rank ordered on a scale that has equal distances between points on the scale).
An appropriate statistic to determine the differences between groups (experimental versus control) is a t-test. An ANOVA is
also appropriate for interval data, and is used for more than two groups. Chi square is appropriate to determine differences
between groups if nominal data (rank ordering with no specific intervals between ranks) is used. Choice C is indicated to
determine correlational relationships.
B166 C8 II
Are highly structured to reduce anxiety and confusion.
Patients with dementia (e.g. Alzheimers disease) benefit from a structured, soothing environment designed to ensure safety
and optimal function. These patients respond poorly to highly stimulating environments or situations requiring greater
cognitive skills than they possess.
B167 C2 III
333687839.doc

Answers

49

Ranging individual muscles according to specific functional needs.


Selective stretching techniques are indicated for the patient with spinal cord injury. Overstretching the long finger flexors
will result in loss of tenodesis grasp. Overstretching the back extensors will result in loss of sitting stability.
B168 C7 III
Have the PTA supervise the class using the outline exercise protocol until the therapist returns.
The PTA can supervise the class using the outline exercise protocol approved by the physical therapist. The PTA should not
change the plan of care.
B169 C1 III
Active eccentric contractions in the painfree range.
Isokinetic, isometric and isotonic exercises do not allow for painfree muscle contractions and can cause further
inflammation of the muscle. For a muscle that is chronically inflamed, focus should be placed on eccentric contractions
since there is less effort and stress placed on the contractile units as compared to concentric contractions at the same level
of work.
B170 C5 III
Decrease the ultrasound intensity.
Acoustical energy is reflected form the bone into the bone-tissue interface resulting in rapid tissue temperature elevation
and stimulation of the highly sensitive periosteum of the bone. A reduction in intensity is indicated if a strong ache is felt.
B171 C7 III
Commission on Accreditation of Rehabilitation Facilities (CARF).
CARF accredits all rehabilitation facilities, including free standing work conditioning centers. OSHA and Dept. of HHS
are not accrediting agencies. JCAHO primarily accredits hospitals, hospices and home care agencies.
B172 C4 II
Report these findings promptly to the physician.
These signs and symptoms are characteristic of heterotopic ossification (HO), an abnormal bone growth typically around a
joint. While the etiology is unknown, its presence can lead to serious ROM limitations. These findings should be reported
promptly to the physician. Massage and ROM exercise could exacerbate the condition. Ice does decrease metabolic
activity; however, more in-depth medical management is required.
B173 C8 I
Spastic diplegic cerebral palsy.
Spastic diplegia includes involvement of all four extremities with the lower extremity involvement being more severe.
B174 C3 I
Tachycardia at a relatively low intensity of exercise with ST segment depression.
The typical exercise ECG changes in the patient with coronary artery disease include rapid HR at very low levels of
exercise intensity. The ST segment becomes depressed (greater than 1 mm is significant) and exertional arrhythmias
(multifocal or runs of PVCs are significant) may appear.
B175 C1 II
Posterior cervical muscle stretching and postural reeducation to relieve vertebral artery compression.
Long-term postural changes with forward head posture include shortening of the posterior muscles with possible vertebral
artery compromise at the occiput.
B176 C6 I
Lateral foot rotation at heel strike.
Bumpers in a single axis prosthetic foot limit motion in either dorsiflexion or plantarflexion. A plantarflexion bumper that
is too stiff can cause lateral foot rotation at heel strike since the prosthetic foot remains dorsiflexed and the foot pivots.

333687839.doc

50

Answers

B177 C3 III
Place a blood pressure cuff on the involved extremity and inflate the cuff until the bleeding stops.
A radial arterial line is a catheter placed in the artery itself. If it becomes dislodged during treatment, the artery is now open
to bleeding. This arterial bleeding needs to be stopped immediately, though it is not considered a cardiac emergency.
Either place a blood pressure cuff above the site of bleeding and inflate the cuff to above systole to stop the bleeding or
place enough manual pressure on the site to stop the bleeding. Then call for help. Elevating the site of bleeding above
heart level will not help as this is an arterial bleed. As long as the heart is pumping with adequate pressure, the site will
continue to bleed. Never replace any line that has come disconnected. The line is no longer sterile and should not be reinserted into the patient. A new sterile catheter will need to be used if the radial line is to be replaced.
B178 C2 I
Poor judgment with increased safety issues.
The patient with left hemiplegia (right CVA) will typically demonstrate poor judgment with numerous safety issues. The
patient with right hemiplegia (left CVA) will demonstrate slow, cautious behaviors and require more feedback and support.
B179 C4 II
Open wounds must be contained within a dressing.
Staphylococcal organisms are spread by contact. Standard germicidal cleaning measures (hand washing) should be
followed. The therapist should be gloved for all direct contact with an open, draining areas. All equipment should be
cleaned with an approved germicidal agent before and after use. Whirlpool may be used. Open wounds must be wellcontained with a dressing. Isolation in a private room is not required.
B180 C1 I
Gall bladder.
Dysfunction of the gall bladder often refers pain to the thorax. The commonly observed referral pattern of the heart is to
the chest and upper extremity, the ovaries to the low back, and the appendix to the right lower quadrant.
B181 C6 III
Allows limited sagittal plane motion with a small amount of mediolateral motion.
The SACH foot (solid ankle cushion heel) is the most commonly prescribed type of prosthetic foot. It provides for sagittal
plane motion (primarily plantarflexion) and very limited frontal plane motion (mediolateral motion).
B182 C8 II
Look at his feet for placement while walking.
Selection of compensatory strategies for sensory losses is dependent upon careful assessment of the sensory systems
contributing to balance (somatosensory, visual, vestibular). Control should be refocused to use available intact sensory
systems. In this case proprioception is impaired while vision is intact. Use of counting can aid gait rhythm but not foot
placement. Light touch down support can aid balance but training using available furniture is a bad idea.
B183 C3 I
Right hemothorax
A left pneumothorax and a left pleural effusion take up space in the left thorax. The air (pneumothorax) or the sterile fluid
(effusion) in the pleural space would push contents of the left hemithorax, including the trachea, to the right. A lung
collapse, or a volume loss phenomenon, on the right would pull the trachea over towards the right. A right hemothorax
(blood was in the pleural space) takes up space in the right hemithorax, shifting the trachea to the left.
B184 C5 II
Gate control mechanisms.
The gate control mechanism is activated by the application of conventional (high rate) TENS, whereas low rate TENS,
having a stronger stimulus and a longer pulse duration activates the ascending inhibition and endorphin production
mechanisms.
B185 C1 II
Secondary gain.
A patient who is able to drive to the clinic for treatment and relates a pain level of 10/10 is not providing consistent
subjective data. Secondary gain in this case, no working, is a likely factor.

333687839.doc

Answers

51

B186 C3 I
Be low at rest and rise very little with exercise.
Beta-adrenergic blocking agents (e.g., propranolol/Inderal) are used to treat hypertension, prevent angina pectoris and
prevent certain arrhythmias. In individuals taking these drugs, heart rate is low at rest and rises very little with exercise
(blunted response). These changes therefore invalidate the use of HR to monitor exercise responses. A more sensitive
measure would RPE (Ratings of Perceived Exertion).
B187 C7 III
Speak to the supervisor about concerns that this policy conflicts with the Code of Ethics.
Although the instructions given by the supervisor may place an individual therapist in conflict with the professions Code of
Ethics, it is possible that the supervisor is unaware of the principle. Speaking with the supervisor instead of breaching the
Code may facilitate a collegial dialogue leading to appropriate changes in policy in the clinic.
B188 C4 III
Use daily hot soaks and moisturize the skin.
Daily hot soaks are contraindicated because of the increased risk of thermal injury. The patient with diabetes typically has
loss of protective sensations and should be taught techniques of foot care. All other interventions are correct advice.
B189 C1 I
Pain provoked with passive glenohumeral external rotation.
The subscapularis is an internal rotator of the humerus. It will be painful if passively stretched into external rotation and
irritated when contracting or being resisted when the shoulder internally rotates. The muscle inserts onto the lesser
tuberosity of the humerus and play no role in shoulder adduction.
B190 C1 II
Closed-chain lower extremity strengthening and proprioceptive exercises.
The most effective treatment for this athlete would involve closed-chain exercises and proprioceptive training, appropriate
interventions for early subacute phase management. The other choices are not appropriate or timely for early subacute
phase management.
B191 C8 II
High detection sensitivity is needed to detect low amplitude signals generated by a low number of motor units such as in a
weak extensor carpi radialis longus. Wide electrode placement would pick up signals from more than one muscle and
might invalidate the procedure.
B192 C8 I
Weight loss and social withdrawal.
Depression is associated with symptoms of withdrawal, fatigue and weight loss. The Geriatric Depression Scale is a valid
measure of depression in the elderly. High, not low scores (above 8 out of a possible 30) are indicative of depression.
Sleep apnea is a potentially lethal disorder in which breathing stops to 10 seconds or more, many times a night. It is
associated with obesity and anatomical obstruction.
B193 C7 III
Medicaid.
Medicaid (a federal-state partnership) covers health care for the indigent. Medicare is a federal program providing care for
the elderly (65 and over) or for certain chronic illnesses. Social security does not provide funds for healthcare.
B194 C3 I
Any swelling in the calf or ankle.
Deep vein thrombophlebitis is characterized by signs of inflammation (tenderness, aching, swelling) typically in the calf.
Tachycardia may be present, not bradycardia. Slight fever can be present, as part of the inflammatory reaction, not lowered
temperature. Homans sign is pain in the calf perceived with squeezing the calf and dorsiflexion and is no longer
considered reliable and lacks specificity and sensitivity. Rapid screening is possible with Doppler ultrasonography while
venous duplex scanning is the primary diagnostic test for DVT.
B195 C1 I
Disc displacement.
The jaw becomes locked in an open position when the disc is displaced. The muscles influence lateral deviation of the jaw
with opening. The retrodiscal lamina cannot get entrapped nor can the temporomandibular ligament become impinged.
333687839.doc

52

Answers

B196 C4 II
Document the findings and consult with the surgeon immediately following treatment.
This patient is most likely experiencing jaundice as a result of liver dysfunction. The therapists best course of action is to
document the findings and consult with her surgeon immediately, preferably by phone.
B197 C1 II
Patient education regarding positions and movements to avoid.
Patients with cemented total hip replacements should initially be weight bearing to tolerance using a walker. Education
regarding positions and movements to avoid is the number one priority. Standard hip precautions stress avoiding excessive
flexion, internal rotation, and adduction.
B198 C7 III
Responsible for communicating findings to the referring physician when the findings are inconsistent with a referring
physicians diagnosis.
When a referral relationship exists with another health care professional, it is the physical therapists responsibility to
communicate with the referring practitioner regarding the physical therapy examination, treatment plan and management of
the referred patient. This is particularly crucial when the findings are inconsistent with the referrers diagnosis.
B199 C5 III
Decongesting the proximal portions of the limb first and working distally.
Lymphedema is a swelling of the soft tissues that occurs with an accumulation of protein-rich fluid in the extracellular
spaces. Causes of primary lymphedema include developmental abnormalities, heredity or unknown etiology. Stage II
lymphedema is characterized by nonpitting edema with connective scar tissue and clinical fibrosis. Lymphatic drainage is
assisted by manual stroking (e.g., Vodder, Leduc, Foldi, Casley-Smith pressure techniques). All techniques use cardial
principles: proximal limb segments before distal, trunk segments before limb segments, and directing the flow of the
lymphatics centrally toward the lymphatic ducts.
B200 C6 II
Tilt-in-space wheelchair with a pelvic belt.
A patient with strong extensor tone needs controls over the hips (pelvic belt) to maintain the hips in flexion. The tilt-in
space design also assists in keeping the patient from coming out of the chair when extensor spasms are active. The other
choices do not adequately address these problems.

333687839.doc

You might also like